Re: [obm-l] Re: [obm-l] Dúvida conceitual (equações)

2018-10-14 Por tôpico Jefferson Franca
 Concordo com Pedro
Em domingo, 14 de outubro de 2018 19:51:25 BRT, Pedro José 
 escreveu:  
 
 Boa noite!Bom questionamento. Vou me posicionar na arquibancada. Minha posição 
é controversa. Se quer se levar em conta a repetição tem que se falar do 
produto das raízes, cada elevada a sua multiplicidade. No caso de soma, cada 
raiz multiplicada pela multiplicidade.Para esse exemplo, o conjunto solução é 
{1/2,-1} então o produto é -1/2.Em suma, não aceito n raízes iguais, mas sim 
uma raiz de multiplicidade n.Se quando queremos provar que algo é unico supomos 
a existência de dois e provamos que são iguais. Creio que seja contraditório 
dois ou nais iguais.Mas vamos observar as diversas posições, pois, creio que o 
assunto não seja pacífico. Saudações, PJMS 
Em Dom, 14 de out de 2018 06:33, Vanderlei Nemitz  
escreveu:

Bom dia!Na seguinte questão, que me foi apresentada por um aluno, a resposta 
proposta é a alternativa C (1/2). Eu sempre pensei que apenas considerávamos 
multiplicidades em equações polinomiais. Como essa é uma equação exponencial, 
obtive a resposta B (-1/2). O que é correto pensar?
O produto das raízes da equação 16.4^3x - 40.4^2x + 17.4^x - 2 = 0 é igual a:A) 
1B) - 0,5C) 0,5D) - 1E) 0
Muito obrigado!
--
Esta mensagem foi verificada pelo sistema de antivírus e 
 acredita-se estar livre de perigo.

--
Esta mensagem foi verificada pelo sistema de antiv�rus e 
 acredita-se estar livre de perigo.  
-- 
Esta mensagem foi verificada pelo sistema de antiv�rus e
 acredita-se estar livre de perigo.



[obm-l] Função Sobrejetiva

2018-03-12 Por tôpico Jefferson Franca
Bom dia pessoal. Estava estudando e um vizinho me propôs o seguinte problema: 
"Suponha que existe uma função f:N -> N tal que f(2n + f(n)) = n. Prove que f é 
sobrejetiva." Pensei em estudar f para n = 0, 1 e - 1 , mas não obtive nada de 
interessanteAlguém poderia me ajudar, por favor?AttJefferson
-- 
Esta mensagem foi verificada pelo sistema de antiv�rus e
 acredita-se estar livre de perigo.



Re: [obm-l] Re: [obm-l] Re: [obm-l] Re: [obm-l] Re: [obm-l] Re: [obm-l] PDF sobre OLimpíadas

2016-06-29 Por tôpico Jefferson Franca
Prezado Israel, simplesmente espetacular!Muito obrigado 

Em Terça-feira, 28 de Junho de 2016 15:00, Israel Meireles Chrisostomo 
 escreveu:
 

 Obrigado, muito importante o reconhecimento de vcs!
Em 28 de junho de 2016 13:21,  escreveu:

Caro Israel:
Excelente. Parabéns pelo incrível trabalho.
Os jovens estudantes olímpicos vão adorar.
Abraço,
E. Wagner.


Quoting Sergio Lima :


Caro Israel,

Dizer que o trabalho é ótimo, incrível e espetacular é até pouco.
O trabalho é surreal. O esforço deve ter sido descomunal, certamente
fruto de anos de trabalho, digitação, diagramação, preparação das figuras
etc.

Guardei para saborear com calma ao londo dos próximos dias, meses e anos.

Abraço,
sergio




2016-06-24 21:47 GMT-03:00 Israel Meireles Chrisostomo <
israelmchrisost...@gmail.com>:


Obrigado gente!

Em 24 de junho de 2016 20:32, Carlos Gomes  escreveu:


Acabei de folhear rapidamente, mas mesmo rápido já dá para perceber a
qualidade do seu trabalho Israel. Parabéns, a comunidade olímpica agradece!

Abraço, Cgomes.

Em 24 de junho de 2016 20:13, Mauricio de Araujo <
mauricio.de.ara...@gmail.com> escreveu:


Israel, muito bom este trabalho!! vou dar uma olhada e, se for o caso,
sugerirei alguma adequação... parabéns!!

Em 24 de junho de 2016 19:25, Israel Meireles Chrisostomo <
israelmchrisost...@gmail.com> escreveu:


Olá pessoal estou compartilhado um PDF que escrevi, acrescentei vários
problemas:

http://media.wix.com/ugd/3eea37_3049c428c55948f2b8bb069834275f50.pdf

Quem tiver alguma sugestão ou correção, por favor envie para o meu
email, pois muitas pessoas podem se beneficiar com o acerto ou mesmo se
prejudicar com o erro.

Obrigado.

israelmchrisost...@gmail.com

--
Esta mensagem foi verificada pelo sistema de antivírus e
acredita-se estar livre de perigo.





--

Abraços,
oɾnɐɹɐ ǝp oıɔıɹnɐɯ


--
Esta mensagem foi verificada pelo sistema de antivírus e
acredita-se estar livre de perigo.




--
Esta mensagem foi verificada pelo sistema de antivírus e
acredita-se estar livre de perigo.




--
Esta mensagem foi verificada pelo sistema de antivírus e
acredita-se estar livre de perigo.


--
Esta mensagem foi verificada pelo sistema de antiv�rus e
 acredita-se estar livre de perigo.






This message was sent using IMP, the Internet Messaging Program.



-- 
Esta mensagem foi verificada pelo sistema de antivírus e
acredita-se estar livre de perigo.


=
Instru�ões para entrar na lista, sair da lista e usar a lista em
http://www.mat.puc-rio.br/~obmlistas/obm-l.html
=



--
Esta mensagem foi verificada pelo sistema de antiv�us e 
 acredita-se estar livre de perigo.

  
-- 
Esta mensagem foi verificada pelo sistema de antiv�rus e
 acredita-se estar livre de perigo.



Re: [obm-l] divisibilidade

2015-04-08 Por tôpico Jefferson Franca
Muito obrigado. Tentei separar os números de outra forma, talvez por isso não 
tenha enxergado outro caminho. Vacilo!Novamente obrigado Esdras.AttJefferson 


 Em Quarta-feira, 8 de Abril de 2015 16:24, Esdras Muniz 
esdrasmunizm...@gmail.com escreveu:
   

 999+1999000=11998999 =12x10⁶-1001=12x10⁶+3000-4000+1=(3000-1)(4000+1).
Em 8 de abril de 2015 12:04, Jefferson Franca jeffma...@yahoo.com.br escreveu:

Fiquei boa parte da madrugada tentando desvendar esse santo mistério: Mostre 
que o número 999+ 1999000 não é primo, ou seja, é composto.Será que alguém 
sabe como resolver esse problema interessante?AttJefferson
--
Esta mensagem foi verificada pelo sistema de antivírus e 
 acredita-se estar livre de perigo.



-- 
Esdras Muniz Mota
Mestrando em Matemática
Universidade Federal do Ceará



--
Esta mensagem foi verificada pelo sistema de antiv�us e 
 acredita-se estar livre de perigo.

  
-- 
Esta mensagem foi verificada pelo sistema de antiv�rus e
 acredita-se estar livre de perigo.



[obm-l] divisibilidade

2015-04-08 Por tôpico Jefferson Franca
Fiquei boa parte da madrugada tentando desvendar esse santo mistério: Mostre 
que o número 999+ 1999000 não é primo, ou seja, é composto.Será que alguém 
sabe como resolver esse problema interessante?AttJefferson
-- 
Esta mensagem foi verificada pelo sistema de antiv�rus e
 acredita-se estar livre de perigo.



[obm-l] Sequências

2015-02-25 Por tôpico Jefferson Franca
Boa tarde para todos. Um aluno me enviou este problema que não consigo 
resolver: Juquinha gosta de diversões matemáticas, uma delas consiste em 
descobrir números de sequências. Por exemplo, 
1,2,2,3,3,3,4,4,4,4,5,5,5,5,5,..., onde cada número natural n é escrito n 
vezes. Determine o número de ordem 1000.Será que alguém aqui saberia elucidar 
este mistério?AttJefferson
-- 
Esta mensagem foi verificada pelo sistema de antiv�rus e
 acredita-se estar livre de perigo.



Re: [obm-l] Re: [obm-l] Sequências

2015-02-25 Por tôpico Jefferson Franca
Obrigado Vanderlei 

 Em Quarta-feira, 25 de Fevereiro de 2015 17:05, Vanderlei Nemitz 
vanderma...@gmail.com escreveu:
   

 Como cada número n aparece n vezes, vamosprocurar o maior valor de n tal que 1 
+ 2 + 3 + ... + n  1000.Assim:(1 + n)·n/2  1000 ⇒ n·(n + 1)  2000O maior 
valor de n que satisfaz adesigualdade anterior é n = 44Assim, após escrevermos 
os 44 números44, teremos escrito (1 + 44)·45/2 = 990 números. Portanto, o 
número de ordem1000 é 45, pois será escrito 45 vezes. Se a pergunta fosse o 
algarismo de ordem 1000, a resposta seria outra.
Em 25 de fevereiro de 2015 16:09, Jefferson Franca jeffma...@yahoo.com.br 
escreveu:

Boa tarde para todos. Um aluno me enviou este problema que não consigo 
resolver: Juquinha gosta de diversões matemáticas, uma delas consiste em 
descobrir números de sequências. Por exemplo, 
1,2,2,3,3,3,4,4,4,4,5,5,5,5,5,..., onde cada número natural n é escrito n 
vezes. Determine o número de ordem 1000.Será que alguém aqui saberia elucidar 
este mistério?AttJefferson
--
Esta mensagem foi verificada pelo sistema de antivírus e 
 acredita-se estar livre de perigo.


--
Esta mensagem foi verificada pelo sistema de antiv�us e 
 acredita-se estar livre de perigo.

   
-- 
Esta mensagem foi verificada pelo sistema de antiv�rus e
 acredita-se estar livre de perigo.



Re: [obm-l] Apostila de Desenho 2 Impacto OFF TOPIC

2013-12-03 Por tôpico Jefferson Franca
Muito obrigado



Em Terça-feira, 3 de Dezembro de 2013 17:36, jjun...@fazenda.ms.gov.br 
jjun...@fazenda.ms.gov.br escreveu:
 
Prezados amigos, 

Não há necessidade de reembolso. 
Os valores gastos não foram elevados. 
Vocês serão úteis a vários e distintos jovens pelo país, e, assim, eu também, 
indiretamente...

Fraternalmente, João.




- Mensagem Original -

De:
obm-l@mat.puc-rio.br

Para:obm-l@mat.puc-rio.br obm-l@mat.puc-rio.br

Cópia:~e

Enviado:Tue, 3 Dec 2013 11:57:08 -0800 (PST)

Assunto:Re: [obm-l] Apostila de Desenho 2 Impacto OFF TOPIC



Obrigado João pelo envio do material.
Me informe o numero e agencia de sua conta para que eu possa lhe pagar os 
custos de xerox e correio (e mais tempo perdido para fazer o serviço).
Mais uma vez agradeço e o que precisar é só avisar.
Abraços
Graciliano



Em Terça-feira, 3 de Dezembro de 2013 15:43, Carlos Victor 
victorcar...@globo.com escreveu:
 
Obrigado João,


Envie-me a sua conta bancária para depósito, ok ?


Também posso lhe enviar o custo pelo correio.


Agradeço e fico a disposição para o que precisares.


Abraços


Carlos Victor



Em 1 de dezembro de 2013 17:54, jjun...@fazenda.ms.gov.br escreveu:

 Senhores: 


Ontem (sábado), por volta das 15h em Campo Grande, foi enviada cópia da 
Apostila 2 de Desenho do IMPACTO, uma ao senhor Carlos Victor (Nilópolis - 
RJ), e outra a Graciliano Antônio Damazo (Penápolis - SP).


ATT.
João (Campo Grande - MS)

-- 
Esta mensagem foi verificada pelo sistema de antivírus e 
acredita-se estar livre de perigo. 

-- 
Esta mensagem foi verificada pelo sistema de antivírus e 

acredita-se estar livre de perigo. 


-- 
Esta mensagem foi verificada pelo sistema de antivírus e 
acredita-se estar livre de perigo. 
-- 
Esta mensagem foi verificada pelo sistema de antiv�us e 
acredita-se estar livre de perigo. 
-- 
Esta mensagem foi verificada pelo sistema de antiv�rus e
 acredita-se estar livre de perigo.



[obm-l] Re: [obm-l] Re: [obm-l] Aritmética não tão básica!

2013-11-28 Por tôpico Jefferson Franca
Perdão, mas não conseguir entender pq os números têm que ser quadrados 
perfeitos ou ter expoente maior que 2?
Vc poderia explicar melhor?
Obrigado
Jefferson



Em Terça-feira, 26 de Novembro de 2013 19:07, saulo nilson 
saulo.nil...@gmail.com escreveu:
 
p+a^2= x^2 numeros da forma quadratica ou cujo expoente e par maior que 2.
p+a^2=x^2n
p=(x^n-a)(x^n+a) absurdo pois p e primo



2013/11/25 Jefferson Franca jeffma...@yahoo.com.br

Estudando surgiram algumas dúvidas. Diante disso, peço humildemente vossa 
ajuda. Eis as dúvidas:
01. Mostre que para um determinado tipo de números a conjectura não é 
verdadeira:'' Todo inteiro positivo pode ser escrito da forma p + a^2 , onde p 
é um número primo ou 1 e a = 0.
02. Ache o número primo p que satisfaz p^6 + 3 =(côngruo)1 (mod p+2).
Att
Jefferson
-- 
Esta mensagem foi verificada pelo sistema de antivírus e 
acredita-se estar livre de perigo. 

-- 
Esta mensagem foi verificada pelo sistema de antivírus e 
acredita-se estar livre de perigo. 
-- 
Esta mensagem foi verificada pelo sistema de antivírus e
 acredita-se estar livre de perigo.



[obm-l] Re: [obm-l] RE: [obm-l] Re: [obm-l] Re: [obm-l] Aritmética não tão básica!

2013-11-28 Por tôpico Jefferson Franca
Muito obrigado Saulo.
Jefferson



Em Quarta-feira, 27 de Novembro de 2013 12:01, marcone augusto araújo borges 
marconeborge...@hotmail.com escreveu:
 
Para o segundo,eu achei p = 31
p6  + 2 = 0(mod(p+2))
p6 + 2 = k(p+2)
Dividindo p6 + 2 por p+2, verifiquei que
k = (p6 + 2)/(p+2) = Q(p) + 66/(p+2)
como k é inteiro e Q(p)  também,temos que
(p+2) divide 66,então p = 31



Date: Tue, 26 Nov 2013 19:53:35 -0800
From: jeffma...@yahoo.com.br
Subject: [obm-l] Re: [obm-l] Re: [obm-l] Aritmética não tão básica!
To: obm-l@mat.puc-rio.br


Obrigado Saulo



Em Terça-feira, 26 de Novembro de 2013 19:07, saulo nilson 
saulo.nil...@gmail.com escreveu:
 
p+a^2= x^2 numeros da forma quadratica ou cujo expoente e par maior que 2.
p+a^2=x^2n
p=(x^n-a)(x^n+a) absurdo pois p e primo



2013/11/25 Jefferson Franca jeffma...@yahoo.com.br

Estudando surgiram algumas dúvidas. Diante disso, peço humildemente vossa 
ajuda. Eis as dúvidas:
01. Mostre que para um determinado tipo de números a conjectura não é 
verdadeira:'' Todo inteiro positivo pode ser escrito da forma p + a^2 , onde p 
é um número primo ou 1 e a = 0.
02. Ache o número primo p que satisfaz p^6 + 3 =(côngruo)1 (mod p+2).
Att
Jefferson
-- 
Esta mensagem foi verificada pelo sistema de antivírus e 
acredita-se estar livre de perigo. 

-- 
Esta mensagem foi verificada pelo sistema de antivírus e 
acredita-se estar livre de perigo. 



-- 
Esta mensagem foi verificada pelo sistema de antivírus e 
acredita-se estar livre de perigo.

-- 
Esta mensagem foi verificada pelo sistema de antivírus e 
acredita-se estar livre de perigo. 
-- 
Esta mensagem foi verificada pelo sistema de antivírus e
 acredita-se estar livre de perigo.



[obm-l] Re: [obm-l] Re: [obm-l] Aritmética não tão básica!

2013-11-26 Por tôpico Jefferson Franca
Obrigado Saulo



Em Terça-feira, 26 de Novembro de 2013 19:07, saulo nilson 
saulo.nil...@gmail.com escreveu:
 
p+a^2= x^2 numeros da forma quadratica ou cujo expoente e par maior que 2.
p+a^2=x^2n
p=(x^n-a)(x^n+a) absurdo pois p e primo



2013/11/25 Jefferson Franca jeffma...@yahoo.com.br

Estudando surgiram algumas dúvidas. Diante disso, peço humildemente vossa 
ajuda. Eis as dúvidas:
01. Mostre que para um determinado tipo de números a conjectura não é 
verdadeira:'' Todo inteiro positivo pode ser escrito da forma p + a^2 , onde p 
é um número primo ou 1 e a = 0.
02. Ache o número primo p que satisfaz p^6 + 3 =(côngruo)1 (mod p+2).
Att
Jefferson
-- 
Esta mensagem foi verificada pelo sistema de antivírus e 
acredita-se estar livre de perigo. 

-- 
Esta mensagem foi verificada pelo sistema de antivírus e 
acredita-se estar livre de perigo. 
-- 
Esta mensagem foi verificada pelo sistema de antivírus e
 acredita-se estar livre de perigo.



[obm-l] Aritmética não tão básica!

2013-11-24 Por tôpico Jefferson Franca
Estudando surgiram algumas dúvidas. Diante disso, peço humildemente vossa 
ajuda. Eis as dúvidas:
01. Mostre que para um determinado tipo de números a conjectura não é 
verdadeira:'' Todo inteiro positivo pode ser escrito da forma p + a^2 , onde p 
é um número primo ou 1 e a = 0.
02. Ache o número primo p que satisfaz p^6 + 3 =(côngruo)1 (mod p+2).
Att
Jefferson
-- 
Esta mensagem foi verificada pelo sistema de antivírus e
 acredita-se estar livre de perigo.



[obm-l] Re: [obm-l] RE: [obm-l] RE: [obm-l] Ajuda em Geometria analítica

2013-09-12 Por tôpico Jefferson Franca
Sem dúvida uma solução extremamente elegante. Parabéns!



 De: marcone augusto araújo borges marconeborge...@hotmail.com
Para: obm-l@mat.puc-rio.br obm-l@mat.puc-rio.br 
Enviadas: Quinta-feira, 12 de Setembro de 2013 11:58
Assunto: [obm-l] RE: [obm-l] RE: [obm-l] Ajuda em Geometria analítica
 


 
Ajudou bastante.




From: profmar...@hotmail.com
To: obm-l@mat.puc-rio.br
Subject: [obm-l] RE: [obm-l] Ajuda em Geometria analítica 
Date: Thu, 12 Sep 2013 03:04:48 +

 
Se não houver imperiosidade de usar Geometria Analítica, pode-se empregar, tão 
somente, a propriedade reflexiva da elipse, segundo a qual: uma reta tangente a 
uma elipse por um de seus pontos forma ângulos congruentes com os raios vetores 
referentes a tal ponto.
Desse modo, sejam F e F' os focos da elipse, O seu centro e AB um diâmetro 
qualquer (A e B pertencentes à cônica). Como O é um centro de simetria, AF = 
BF' e AF' = BF. Portanto, AFBF' é um paralelogramo, com diagonais 
encontrando-se em O. Das congruências entre os triângulos AFO e BF'O, bem como 
entre AF'O e BOF, fica fácil ver, usando a propriedade reflexiva, que as retas 
tangentes formam, por exemplo, alternos internos de mesma medida, relativamente 
à reta transversal AOB. Logo, devem ser paralelas.
Obviamente, convém acompanhar a resolução usando uma figura.
Espero ter ajudado.




From: marconeborge...@hotmail.com
To: obm-l@mat.puc-rio.br
Subject: [obm-l] Ajuda em Geometria analítica
Date: Thu, 12 Sep 2013 02:34:54 +

 
Prove que duas retas tangentes a uma elipse pelos pontos extremos de um 
 
diâmetro são paralelas.
-- 
Esta mensagem foi verificada pelo sistema de antivírus e 
acredita-se estar livre de perigo.
-- 
Esta mensagem foi verificada pelo sistema de antivírus e 
acredita-se estar livre de perigo.
-- 
Esta mensagem foi verificada pelo sistema de antivírus e 
acredita-se estar livre de perigo. 
-- 
Esta mensagem foi verificada pelo sistema de antiv�rus e
 acredita-se estar livre de perigo.



[obm-l] Re: [obm-l] Re: [obm-l] Re: [obm-l] Re: [obm-l] Re: [obm-l] Re: [obm-l] problema de geometria difícil

2012-12-30 Por tôpico Jefferson Franca
Tem saída por trigonometria sim, lei dos senos e depois fatoração trigonométrica



 De: felipe araujo costa faraujoco...@yahoo.com.br
Para: obm-l@mat.puc-rio.br obm-l@mat.puc-rio.br 
Enviadas: Quinta-feira, 27 de Dezembro de 2012 16:07
Assunto: [obm-l] Re: [obm-l] Re: [obm-l] Re: [obm-l] Re: [obm-l] Re: [obm-l] 
problema de geometria difícil
 

A solução é por geometria plana.
 
Felipe Araujo Costa
Cel: 77430066
E-mail: faraujoco...@yahoo.com.br
faco...@metalmat.ufrj.br



 De: Vanderlei * vanderma...@gmail.com
Para: obm-l@mat.puc-rio.br 
Enviadas: Domingo, 23 de Dezembro de 2012 20:22
Assunto: [obm-l] Re: [obm-l] Re: [obm-l] Re: [obm-l] Re: [obm-l] problema de 
geometria difícil
 

Obrigado! Quando puder postar, ficarei esperando...


Em 23 de dezembro de 2012 17:31, terence thirteen peterdirich...@gmail.com 
escreveu:

Cara, não tem muito o que fazer. Apliquei trigonometria para obter uma equação 
entre o seno de (10+x) e o seno de x. Depois é fazer soma e subtração de arcos, 
e com um pouco de perspicácia obter uma relação como sen x = sen y. Mas daqui a 
pouco eu passo os detalhes dela - você me pegou desprevenido agora :)




Em 22 de dezembro de 2012 20:10, Vanderlei * vanderma...@gmail.com escreveu:


A resposta está certa! Tem a solução?
Valeu!!!


Em 22 de dezembro de 2012 19:34, terence thirteen peterdirich...@gmail.com 
escreveu:


Trigonometria...

Com um tanto de força,. acho que deu 20. Mas vou reconferir!




Em 21 de dezembro de 2012 00:21, Vanderlei * vanderma...@gmail.com 
escreveu:


Pessoal, alguém conhece uma solução para o primeiro problema da página a 
seguir???


http://thinkzone.wlonk.com/MathFun/Triangle.htm


Ele é chamada de problema mais difícil com geometria fácil do mundo.


O segundo problema é famoso, mas o primeiro...


-- 
/**/
神が祝福

Torres 



-- 
/**/
神が祝福

Torres 

Re: [obm-l] RE: PROBLEMA

2012-12-09 Por tôpico Jefferson Franca
Eu tbm compartilho do mesmo problema, ou seja, não tenho conseguido receber e 
nem enviar emails pra esta lista. Alguém sabe o pq disto?
Abs



 De: Luís Lopes qed_te...@hotmail.com
Para: obm-l@mat.puc-rio.br obm-l@mat.puc-rio.br 
Enviadas: Quarta-feira, 5 de Dezembro de 2012 11:16
Assunto: [obm-l] RE: PROBLEMA
 

 
Sauda,c~oes, 

Não tenho recebido e conseguido mandar msgs para a lista. 

Esta vai como teste. Se receberem, continuem a ler. 

Luís 




From: qed_te...@hotmail.com
To: obm-l@mat.puc-rio.br
Subject: FW: PROBLEMA
Date: Mon, 3 Dec 2012 12:44:01 +

 
Sauda,c~oes, 

Mais uma vez, peço a ajuda de vocês para uma resposta 
mais completa e interessante. 

Espero que o anexo passe. Se não, alguém poderia colocá-lo 
disponível em algum site? 



Subject: PROBLEMA
Date: Mon, 3 Dec 2012 08:17:24 -0300

 
Prezado Luís, 


Gostaria de sua ajuda para o problema(conforme 
arquivo PDF anexo). 
Se possível, gostaria que justificasse porque os demais 
itens são verdadeiros. 


Abraços, 
FERNANDO FORTALEZA-CE. 

PS: E eu gostaria de saber também por que o item falso é falso. 
Obrigado. 

Abraços, 
Luís 

[obm-l] geometria analítica

2012-11-06 Por tôpico Jefferson Franca
Olá ! Realmente esta questão está tirando meu sono. Será que alguém pode ter 
uma iluminação divina e me ajudar?
Seja M um ponto de uma elipse com
centro O e focos F1 e F2. A reta r é tangente à elipse no
ponto M e s é uma reta , que passa por O, paralela a r. As retas suportes dos
raios vetores MF1 e MF2 intersectam a reta s em H1 e H2, respectivamente. 
Sabendo que o segmento F1H1 mede 2 cm, calcule o comprimento de F2H2.
Abs

[obm-l] aritmética

2012-04-15 Por tôpico Jefferson Franca
Um aluno muito curioso e estudioso(tomara!) me deu esta questão durante uma 
aula semana passada e tentei, tentei e nada!
Será que alguém pode dar um ajuda aí?
Em
uma base R1 uma fração F1 se escreve como 0,373737... enquanto que uma
fração F2 é escrita como0,737373 . Em outra base R2, a fração F1 é escrita como 
0,252525...  e a fração F2 como 0,525252...A soma R1 + R2 no sistema de
numeração decimal é:
a)
24   b)
22  c) 21   d)
20    e) 19

Re: [obm-l] Polos Olimpicos de Treinamento

2012-04-15 Por tôpico Jefferson Franca
Espetacular essa iniciativa. Todos os envolvidos estão de parabéns!



 De: samuel barbosa samuelb...@gmail.com
Para: obm-l@mat.puc-rio.br 
Enviadas: Domingo, 25 de Março de 2012 15:39
Assunto: [obm-l] Polos Olimpicos de Treinamento
 

Oi pessoal,

Um esforço coletivo da OBMEP, IMPA e OBM está criando um programa chamado polos 
olímpicos de treinamento que disponibilizará aulas em vídeo e materiais 
teóricos em texto com a intenção de popularizar o acesso à treinamento de 
qualidade no Brasil. Vale a pena conferir:

http://www.pot.impa.br/

Abraços
Samuel

[obm-l] Re: [obm-l] Valor máximo e mínimo

2012-02-22 Por tôpico Jefferson Franca
Boa noite.
Acredito que vc deva estudar essa equação do segundo grau em função de x ou y, 
ou seja, ache em função de y ou vice-versa, e depois analise as raízes.
abs



 De: João Maldonado joao_maldona...@hotmail.com
Para: obm-l@mat.puc-rio.br 
Enviadas: Segunda-feira, 20 de Fevereiro de 2012 21:23
Assunto: [obm-l] Valor máximo e mínimo
 

 
Se a e b são respectivamente os valores máximos mínimos de y/x, com x, y0  que 
satisfazem a quação 2x²+xy + 3y² - 11x - 20y + 40 = 0 então,  o valor de a + b 
é igual a :

a) 3      b) sqrt(10)        c) 7/2       d) 9/2     e) 2sqrt(14)

[obm-l] Re: [obm-l] Re: [obm-l] Questão de geometria plana!! ajuda em nova solução

2011-09-12 Por tôpico Jefferson Franca
Que tal se vc unir este ponto aos vértices do triangulo, formando assim outros 
3 triangulos menores tais que a soma das áreas deles seja a área do triangulo 
equilátero ?
abs e boa sorte




De: Johann Dirichlet peterdirich...@gmail.com
Para: obm-l@mat.puc-rio.br
Enviadas: Sábado, 10 de Setembro de 2011 17:17
Assunto: [obm-l] Re: [obm-l] Questão de geometria plana!! ajuda em nova solução

Ué, deslizar o triângulo pra baixo já é sintético. E é a única que eu
imagino agora.

Em 10/09/11, 
douglas.olive...@grupoolimpo.com.brdouglas.olive...@grupoolimpo.com.br
escreveu:


 Olá boa tarde, estou com uma questão de geometria plana, que diz
 assim: Em um triângulo equilátero, um ponto P interno dista de sues
 vértices 5 , 7, e 8 de sues vértices, achar o lado.

 gostaria de uma
 ajudinha, para elaborar uma nova solução, pois conheco a do oswaldo
 dolce, que transporta um triângulo( esta solucao encontra-se no gabarito
 do livro de matematica elementar numero 9), tambem conheco uma fazendo
 duas leis dos cossenos, e tambem elaborei uma em geometria analitica
 fazendo distancia de ponto a ponto , gostaria de uma ajuda para elaborar
 outra mas totalmente voltada para geometria plana, dede ja agradeco.



-- 
/**/
神���

Torres

=
Instruções para entrar na lista, sair da lista e usar a lista em
http://www.mat.puc-rio.br/~obmlistas/obm-l.html
=

[obm-l] Re: [obm-l] dúvida - teoria dos números

2011-07-31 Por tôpico Jefferson Franca
Tente pensar no pequeno Teorema de Newton, ou se preferir use congruencias.
abs



De: Marco Antonio Leal marcoantonio_elemen...@hotmail.com
Para: obm-l@mat.puc-rio.br
Enviadas: Quinta-feira, 28 de Julho de 2011 21:39
Assunto: [obm-l] dúvida - teoria dos números


 
Não estou conseguindo uma prova satisfatória para o seguinte exercício:
 
prove que 2 ^23  -  1 é divisivel por 47.

[obm-l] Re: [obm-l] dúvida - teoria dos números

2011-07-31 Por tôpico Jefferson Franca
Desculpe, aliás, devo pedir desculpas a Fermat, troquei o Fermat por Newton, ou 
seja, use o pequeno teorema de FERMAT.
Boa sorte aí



De: Marco Antonio Leal marcoantonio_elemen...@hotmail.com
Para: obm-l@mat.puc-rio.br
Enviadas: Quinta-feira, 28 de Julho de 2011 21:39
Assunto: [obm-l] dúvida - teoria dos números


 
Não estou conseguindo uma prova satisfatória para o seguinte exercício:
 
prove que 2 ^23  -  1 é divisivel por 47.

[obm-l] Re: [obm-l] Re: [obm-l] questão trigonometria complicada

2011-07-25 Por tôpico Jefferson Franca
Professor novamente agradeço ao senhor pela resolução VERDADEIRAMENTE 
inteligente e digna de um trabalhador!!!
obrigado e abs




De: Ralph Teixeira ralp...@gmail.com
Para: obm-l@mat.puc-rio.br
Enviadas: Segunda-feira, 27 de Junho de 2011 17:43
Assunto: [obm-l] Re: [obm-l] questão trigonometria complicada


Hmmm, vejamos. Será que a gente arruma algum polinômio cujas raízes sejam as 3 
parcelas da sua soma?
 
Considere a famosa identidade 
trigonométrica sin7t=(8(cos2t)^3+4(cos2t)^2-4(cos2t)-1).sint
 
(Desculpa, não pude resistir.)
 
Note que t=kpi/7 (k=1,2,4) dá três raízes de sin7t, mas nenhum deles dá raiz de 
sint. Então estes valores de t devem anular o termo entre parênteses... Em 
outras palavras, se você considerar o polinômio P(x)=8x^3+4x^2-4x-1, você verá 
que suas raízes são exatamente cos(2pi/7), cos(4pi/7) e cos(8pi/7) -- 
exatamente porque é um polinômio do 3o grau, então se eu achei 3 raízes 
distintas, achei todas.
 
(O argumento também vale para k=3,5,6, mas então obtemos cos(6pi/7)=cos(8pi/7), 
cos(10pi/7)=cos(4pi/7) e cos(12pi/7)=cos(2pi/7), que são aquelas raízes de novo)
 
Em suma, o problema agora é: sejam a,b e c as raízes de P(x)=8x^3+4x^2-4x-1. 
Encontre a^(1/3)+b^(1/3)+c^(1/3).
 
Vou escrever a^(1/3)=A, b^(1/3)=B e c^(1/3)=C. Mas, do polinômio sabemos que
a+b+c=-1/2, isto é, A^3+B^3+C^3=-1/2
ab+ac+bc=1/2, isto é, A^3B^3+A^3C^3+B^3C^3=1/2
abc=-1/8, isto é, ABC=-1/2.
 
Poxa, eu até consigo fazer o resto, mas é HORRENDO. Vamos lá.
 
Agora, talvez você já tenha visto a identidade 
x^3+y^3+z^3-3xyz=(x+y+z)((x+y+z)^2-3(xy+xz+yz))
 
Aplicando esta identidade com (x,y,z)=(A,B,C) temos:
-1/2+3/2=1=S(S^2-3D) (onde S=A+B+C e D=AB+AC+BC)
 
Aplicando esta identidade com (x,y,z)=(AB,AC,BC), temos:
1/2-3(1/4)=-1/4=D(D^2-3SP)=D(D^2+3S/2) (onde P=ABC=-1/2)
 
Enfim, duas equações e duas incógnitas! Tire D da primeira e jogue na segunda 
-- fica horrendo, mas dá uma equação polinomial de grau 9 em S, com termos 
apenas em S^3, S^6 e S^9. Faça S^3=Z, resolva a equação cúbica em Z, S é a raiz 
cúbica de Z.
 
Argh! Tá, fiquei sem vontade de terminar as contas, e devo ter errado algo no 
meio do caminho, mas saiu!
 
Abraço,
   Ralph
 
 
2011/6/26 Jefferson Franca jeffma...@yahoo.com.br

Boa tarde senhores. Será que alguém poderia me iluminar nesta questão: Calcule 
o valor da soma (cos(2*pi/7)^1/3 + (cos(4*pi/7))^1/3 + (cos(8*pi/7))^1/3 ?
abs


[obm-l] Re: [obm-l] Re: [obm-l] questão trigonometria complicada

2011-07-19 Por tôpico Jefferson Franca
Muito, muito , muito obrigado!
Não foi fácil resolver.



De: Ralph Teixeira ralp...@gmail.com
Para: obm-l@mat.puc-rio.br
Enviadas: Segunda-feira, 27 de Junho de 2011 17:52
Assunto: [obm-l] Re: [obm-l] questão trigonometria complicada


Ah, já vi errinho de sinal no meio do caminho, no sinal de ab+ac+bc e no de 
abc. Corrigi abaixo, mas deve haver outros. De qualquer forma, a ideia ainda 
vale.

2011/6/27 Ralph Teixeira ralp...@gmail.com

Hmmm, vejamos. Será que a gente arruma algum polinômio cujas raízes sejam as 3 
parcelas da sua soma?
 
Considere a famosa identidade 
trigonométrica sin7t=(8(cos2t)^3+4(cos2t)^2-4(cos2t)-1).sint
 
(Desculpa, não pude resistir.)
 
Note que t=kpi/7 (k=1,2,4) dá três raízes de sin7t, mas nenhum deles dá raiz 
de sint. Então estes valores de t devem anular o termo entre parênteses... Em 
outras palavras, se você considerar o polinômio P(x)=8x^3+4x^2-4x-1, você verá 
que suas raízes são exatamente cos(2pi/7), cos(4pi/7) e cos(8pi/7) -- 
exatamente porque é um polinômio do 3o grau, então se eu achei 3 raízes 
distintas, achei todas.
 
(O argumento também vale para k=3,5,6, mas então obtemos 
cos(6pi/7)=cos(8pi/7), cos(10pi/7)=cos(4pi/7) e cos(12pi/7)=cos(2pi/7), que 
são aquelas raízes de novo)
 
Em suma, o problema agora é: sejam a,b e c as raízes de P(x)=8x^3+4x^2-4x-1. 
Encontre a^(1/3)+b^(1/3)+c^(1/3).
 
Vou escrever a^(1/3)=A, b^(1/3)=B e c^(1/3)=C. Mas, do polinômio sabemos que
a+b+c=-1/2, isto é, A^3+B^3+C^3=-1/2
ab+ac+bc=-1/2, isto é, A^3B^3+A^3C^3+B^3C^3=-1/2
abc=1/8, isto é, ABC=1/2.
 
Poxa, eu até consigo fazer o resto, mas é HORRENDO. Vamos lá.
 
Agora, talvez você já tenha visto a identidade 
x^3+y^3+z^3-3xyz=(x+y+z)((x+y+z)^2-3(xy+xz+yz))
 
Aplicando esta identidade com (x,y,z)=(A,B,C) temos:
-1/2-3/2=-2=S(S^2-3D) (onde S=A+B+C e D=AB+AC+BC)
 
Aplicando esta identidade com (x,y,z)=(AB,AC,BC), temos:
-1/2-3(1/4)=-5/4=D(D^2-3SP)=D(D^2-3S/2) (onde P=ABC=1/2)
 
Enfim, duas equações e duas incógnitas! Tire D da primeira e jogue na segunda 
-- fica horrendo, mas dá uma equação polinomial de grau 9 em S, com termos 
apenas em S^3, S^6 e S^9. Faça S^3=Z, resolva a equação cúbica em Z, S é a 
raiz cúbica de Z.
 
Argh! Tá, fiquei sem vontade de terminar as contas, e devo ter errado algo no 
meio do caminho, mas saiu!
 
Abraço,
   Ralph
 
 
2011/6/26 Jefferson Franca jeffma...@yahoo.com.br

Boa tarde senhores. Será que alguém poderia me iluminar nesta questão: Calcule 
o valor da soma (cos(2*pi/7)^1/3 + (cos(4*pi/7))^1/3 + (cos(8*pi/7))^1/3 ?
abs



[obm-l] questão trigonometria complicada

2011-06-26 Por tôpico Jefferson Franca
Boa tarde senhores. Será que alguém poderia me iluminar nesta questão: Calcule 
o valor da soma (cos(2*pi/7)^1/3 + (cos(4*pi/7))^1/3 + (cos(8*pi/7))^1/3 ?
abs


[obm-l] questão trigonometria complicada

2011-06-26 Por tôpico Jefferson Franca
Boa tarde senhores. Será que alguém poderia me iluminar nesta questão: Calcule 
o valor da soma (cos(2*pi/7)^1/3 + (cos(4*pi/7))^1/3 + (cos(8*pi/7))^1/3 ?

abs


[obm-l] Re: [obm-l] Re: [obm-l] Re: [obm-l] Soma tória

2010-03-22 Por tôpico Jefferson Franca
Tentei desse jeito que te disse tem 5 min e conseguir.
Boa questão.
POde contar com nosso apoio.
Abs

--- Em dom, 21/3/10, Maycon Maia Vitali mayconm...@yahoo.com.br escreveu:

De: Maycon Maia Vitali mayconm...@yahoo.com.br
Assunto: Re: [obm-l] Re: [obm-l] Re: [obm-l] Somatória
Para: obm-l@mat.puc-rio.br
Data: Domingo, 21 de Março de 2010, 19:14

Fala Jefferson,

Resolvi usando derivação como o Tiago disse. Porém vou fazer dessa forma só 
para praticar e ver se consigo chegar em alguma solução interessante.

Obrigado,
Maycon Maia Vitali

Jefferson Franca escreveu:
 Que tal decompor a somas em várias somas? Por exemplo: escreva em colunas, a 
 primeira linha e primeira coluna x^0 , na segunda linha escreva x^1, na 
 terceira linha escreva x^2 + x^2, na quarta escreva x^3 + x^3 + x^3 , na 
 quinta escreva x^4 + x^4 + x^4 + x^4 e , assim por diante, até a última linha 
 que conterá x^n + x^n + x^n +...+x^n, depois soma as colunas, soma de termos 
 em PG, percebeu? Agora é com vc. Espero ter te ajudado.
 Abs
 
 --- Em *dom, 21/3/10, Tiago /hit0...@gmail.com/* escreveu:
 
 
     De: Tiago hit0...@gmail.com
     Assunto: [obm-l] Re: [obm-l] Somatória
     Para: obm-l@mat.puc-rio.br
     Data: Domingo, 21 de Março de 2010, 15:16
 
     Olá. Não é difícil, mas o único jeito que eu consegui fazer foi
     utilizando derivada. Você sabe derivada? Se sim, tente mais um pouco.
 
     2010/3/21 Maycon Maia Vitali mayconm...@yahoo.com.br
     http://br.mc657.mail.yahoo.com/mc/compose?to=mayconm...@yahoo.com.br
 
         Pessoal,
 
         Tenho o seguinte:
         
http://upload.wikimedia.org/math/d/2/d/d2dff313af2593b914e71cfcacc38ee0.png
 
         Gostaria de saber como consigo, a partir desse somatório chegar
         nessa função. Tentei desenvolver o somatório pra tentar achar
         alguma representação em progressão e nada. E vendo as
         propriedades do somatório não encontrei nada que pudesse utilizar.
 
         Obrigado,
         Maycon
         __
         Faça ligações para outros computadores com o novo Yahoo!
         Messenger http://br.beta.messenger.yahoo.com/
         
=
         Instruções para entrar na lista, sair da lista e usar a lista em
         http://www.mat.puc-rio.br/~obmlistas/obm-l.html
         
=
 
 
 
 
     --     Tiago J. Fonseca
     http://legauss.blogspot.com http://legauss.blogspot.com/
 
 
 
 Veja quais são os assuntos do momento no Yahoo! + Buscados: Top 10 
 http://br.rd.yahoo.com/mail/taglines/mail/*http://br.maisbuscados.yahoo.com/
  - Celebridades 
 http://br.rd.yahoo.com/mail/taglines/mail/*http://br.maisbuscados.yahoo.com/celebridades/
  - Música 
 http://br.rd.yahoo.com/mail/taglines/mail/*http://br.maisbuscados.yahoo.com/m%C3%BAsica/
  - Esportes 
 http://br.rd.yahoo.com/mail/taglines/mail/*http://br.maisbuscados.yahoo.com/esportes/
  

__
Faça ligações para outros computadores com o novo Yahoo! Messenger 
http://br.beta.messenger.yahoo.com/ 
=
Instruções para entrar na lista, sair da lista e usar a lista em
http://www.mat.puc-rio.br/~obmlistas/obm-l.html
=



  

Veja quais são os assuntos do momento no Yahoo! +Buscados
http://br.maisbuscados.yahoo.com

[obm-l] Re: [obm-l] Re: [obm-l] Somatória

2010-03-21 Por tôpico Jefferson Franca
Que tal decompor a somas em várias somas? Por exemplo: escreva em colunas, a 
primeira linha e primeira coluna x^0 , na segunda linha escreva x^1, na 
terceira linha escreva x^2 + x^2, na quarta escreva x^3 + x^3 + x^3 , na quinta 
escreva x^4 + x^4 + x^4 + x^4 e , assim por diante, até a última linha que 
conterá x^n + x^n + x^n +...+x^n, depois soma as colunas, soma de termos em PG, 
percebeu? Agora é com vc. Espero ter te ajudado.
Abs

--- Em dom, 21/3/10, Tiago hit0...@gmail.com escreveu:


De: Tiago hit0...@gmail.com
Assunto: [obm-l] Re: [obm-l] Somatória
Para: obm-l@mat.puc-rio.br
Data: Domingo, 21 de Março de 2010, 15:16


Olá. Não é difícil, mas o único jeito que eu consegui fazer foi utilizando 
derivada. Você sabe derivada? Se sim, tente mais um pouco.


2010/3/21 Maycon Maia Vitali mayconm...@yahoo.com.br

Pessoal,

Tenho o seguinte:
http://upload.wikimedia.org/math/d/2/d/d2dff313af2593b914e71cfcacc38ee0.png

Gostaria de saber como consigo, a partir desse somatório chegar nessa função. 
Tentei desenvolver o somatório pra tentar achar alguma representação em 
progressão e nada. E vendo as propriedades do somatório não encontrei nada que 
pudesse utilizar.

Obrigado,
Maycon

__
Faça ligações para outros computadores com o novo Yahoo! Messenger 
http://br.beta.messenger.yahoo.com/ 
=
Instruções para entrar na lista, sair da lista e usar a lista em
http://www.mat.puc-rio.br/~obmlistas/obm-l.html
=



-- 
Tiago J. Fonseca
http://legauss.blogspot.com



  

Veja quais são os assuntos do momento no Yahoo! +Buscados
http://br.maisbuscados.yahoo.com

[obm-l] Potências

2010-02-11 Por tôpico Jefferson Franca
Será que alguém pode me ajudar com esta questão: Qual a soma dos algarismos do 
número 2^10 + 3^8  + 4^8 + 5^5 + 7^3 ?
Tentei achar algum modo diferente de fazer as contas, porém, não encontrei.
Abs



  

Veja quais são os assuntos do momento no Yahoo! +Buscados
http://br.maisbuscados.yahoo.com

[obm-l] probabilidade

2008-12-15 Por tôpico Jefferson Franca
Ontem, recebi uma questão que ainda não resolvi, será que alguém pode me ajudar?
Lá vai: Um aluno entrevistou sua turma para saber a intenção de votos numa 
pesquisa para representante dela e notou que houve um empate técnico, metade da 
turma votaria no candidato A, enquanto que a outra metade votaria no candidato 
B. Bem, um instituto de pesquisa escolheu aleatoriamente 4 alunos dessa turma, 
qual será a probabilidade desse instituto chegar à mesma conclusão que o aluno?
Obrigado

--- Em sex, 31/10/08, Ralph Teixeira ralp...@gmail.com escreveu:

De: Ralph Teixeira ralp...@gmail.com
Assunto: Re: [obm-l] exercicio simples de probabilidade
Para: obm-l@mat.puc-rio.br
Data: Sexta-feira, 31 de Outubro de 2008, 23:58



Para mim, estao faltando dados... Agora, se voce me disser que:
 
i) Em cada partida, a chance de A vencer eh p;
ii) As partidas sao independentes entre si;
 
Entao (ainda nao estah claro qual eh a pergunta, entao apresento duas 
respostas):
 
Pr(A vencer exatamente 4 partidas) = C(6,4).p^4.(1-p)^2
Pr(A vencer 4 ou mais partidas) = C(6,4).p^4.(1-p)^2+C(6,5).p^5.(1-p)+p^6
 
Em particular, se p=50%, entao:
 
Pr(A vencer exatamente 4) = 15/64 = 23.4375%
Pr(A vencer pelo menos 4) = 11/32 = 34.375%
 
Abraco,
    Ralph


2008/10/31 Graciliano Antonio Damazo bissa_dam...@yahoo.com.br






Caros amigos da lista, tenho uma questao simples de probabilidade que resultou 
numa discussao na resolução da mesma numa aula de reforço que eu estava 
estagiando la vai...mas não vale rirrsrs(brincadeira):
 
1) Dois times A e B jogam 6 partidas entre si. Qual a probabilidade do time A 
vencer 4 dessas partidas?
 
Gostaria de saber como vocês interpretam essa questão. Muito obrigado pela 
atenção desde já.



Novos endereços, o Yahoo! que você conhece. Crie um email novo com a sua cara 
@ymail.com ou @rocketmail.com.



  Veja quais são os assuntos do momento no Yahoo! +Buscados
http://br.maisbuscados.yahoo.com

[obm-l] eleição

2008-12-07 Por tôpico Jefferson Franca
Caros amigos, vai aqui mais uma de eleição que achei bastante interessante e 
ainda não fiz: Um aluno, chamado Rufos, depois de entrevistar todos os seus 
colegas de turma, chegou à conclusão que metade deles votaria no candidato A 
para representante de turma enquanto que a outra metade votaria no candidato B. 
Um instituto de pesquisas escolheu aleatoriamente 4 alunos desta turma. Qual a 
probabilidade deste instituto chegar à mesma conclusão do aluno Rufos?


  Veja quais são os assuntos do momento no Yahoo! +Buscados
http://br.maisbuscados.yahoo.com

Res: [obm-l] Re: Demonstrar por pif...

2007-03-27 Por tôpico Jefferson Franca
Vc já tentou usar a soma telescópica e o fato de que 1/(2n-1)(2n+1) = A/2n-1 + 
B/2n+1?


- Mensagem original 
De: Julio Sousa [EMAIL PROTECTED]
Para: obm-l@mat.puc-rio.br
Enviadas: Segunda-feira, 26 de Março de 2007 22:19:03
Assunto: [obm-l] Re: Demonstrar por pif...

Deduzir as fórmulas das somas:
1) 1/(1*3) + 1/(3*5) + ... + 1/(2n - 1)*(2n + 1)

2) 1/(1*5) + 1/(5*9) + ... + 1/(4n - 3)*(4n + 1)


On 3/26/07, Julio Sousa [EMAIL PROTECTED] wrote:
...que (n^3 + 5n) é divisível por 6

-- 
www.rumoaoita.com
Atenciosamente 
Júlio Sousa 



-- 
Atenciosamente
Júlio Sousa

__
Fale com seus amigos  de graça com o novo Yahoo! Messenger 
http://br.messenger.yahoo.com/ 

[obm-l] Res: [obm-l] Congruência modular

2007-03-27 Por tôpico Jefferson Franca
É só fazer n =2k + 1 ou se vc preferir n = 2k -1.



- Mensagem original 
De: Bruna Carvalho [EMAIL PROTECTED]
Para: obm-l@mat.puc-rio.br
Enviadas: Sábado, 24 de Março de 2007 14:19:14
Assunto: [obm-l] Congruência modular

Se n é ímpar, prove que n²-1 é divisível por 8.
Eu quero aprender como faz esse tipo de questão por congruência, alguém pode me 
dar uma ajudinha.
bjos.

-- 
Bjos, 
Bruna

__
Fale com seus amigos  de graça com o novo Yahoo! Messenger 
http://br.messenger.yahoo.com/ 

[obm-l] Res: [obm-l] Res: [obm-l] Somatório interesa nte..

2006-11-15 Por tôpico Jefferson Franca
Realmente vacilei, não tinha notado que era produtório e não somatório, 
desculpa e obrigado pela dica!
Abs 


- Mensagem original 
De: Iuri [EMAIL PROTECTED]
Para: obm-l@mat.puc-rio.br
Enviadas: Domingo, 12 de Novembro de 2006 16:39:33
Assunto: Re: [obm-l] Res: [obm-l] Somatório interesante..

Essa saida de multiplicar por 2senx só funciona pra produto de cossenos.. 
Multiplicando esse produto por sen2x depois vai cair em sen2x*sen2x, que nao 
ajuda em muita coisa.

Iuri 



On 11/12/06, Jefferson Franca [EMAIL PROTECTED] wrote:
Chame esse somatório de Y, depois multiplique os dois lados por 2sena, note que 
vc terá sempre algo do tipo sen(2x), pronto seus problemas acabaram! 


- Mensagem original 
De: Alex pereira Bezerra  [EMAIL PROTECTED]
Para: obm-l@mat.puc-rio.br
Enviadas: Domingo, 12 de Novembro de 2006 4:31:15 
Assunto: Re: [obm-l] Somatório interesante..


olhe para a fórmula de Euler e separe a parte real da imaginaria,ok

Em 11/11/06, Orlando Onofre Filho [EMAIL PROTECTED] escreveu:



 Olá pessoal . estou precisando de ajuda com o seguinte produtório , qualquer
 ajuda é bem vida.
   sena.sen2a.sen4a.sen8asen2*n=? 

 Obrigado - Orlando

 _
 MSN Messenger: instale grátis e converse com seus amigos.
 http://messenger.msn.com.br

 =
 Instruções para entrar na lista, sair da lista e usar a lista em
 http://www.mat.puc-rio.br/~nicolau/olimp/obm-l.html
 =


=
Instruções para entrar na lista, sair da lista e usar a lista em 
http://www.mat.puc-rio.br/~nicolau/olimp/obm-l.html
= 





Yahoo! Search
Música para ver e ouvir: You're Beautiful, do James Blunt



___ 
Yahoo! Acesso Grátis - Internet rápida e grátis. Instale 
o discador agora! 
http://br.acesso.yahoo.com

[obm-l] Res: [obm-l] Somatório interesante..

2006-11-12 Por tôpico Jefferson Franca
Chame esse somatório de Y, depois multiplique os dois lados por 2sena, note que vc terá sempre algo do tipo sen(2x), pronto seus problemas acabaram!
- Mensagem original De: Alex pereira Bezerra [EMAIL PROTECTED]Para: obm-l@mat.puc-rio.brEnviadas: Domingo, 12 de Novembro de 2006 4:31:15Assunto: Re: [obm-l] Somatório interesante..
olhe para a fórmula de Euler e separe a parte real da imaginaria,okEm 11/11/06, Orlando Onofre Filho[EMAIL PROTECTED] escreveu: Olá pessoal . estou precisando de ajuda com o seguinte produtório , qualquer ajuda é bem vida. sena.sen2a.sen4a.sen8asen2*n=? Obrigado - Orlando _ MSN Messenger: instale grátis e converse com seus amigos. http://messenger.msn.com.br = Instruções para entrar na lista, sair da lista e usar a lista em http://www.mat.puc-rio.br/~nicolau/olimp/obm-l.html
 ==Instruções para entrar na lista, sair da lista e usar a lista emhttp://www.mat.puc-rio.br/~nicolau/olimp/obm-l.html=
		 
Yahoo! Search 
Música para ver e ouvir: You're Beautiful, do James Blunt

[obm-l] Res: [obm-l] Res: [obm-l] Dúvida (Funç ão e Divisibilidade)

2006-10-10 Por tôpico Jefferson Franca
E muito. Valeu!
- Mensagem original De: Qwert Smith [EMAIL PROTECTED]Para: obm-l@mat.puc-rio.brEnviadas: Sábado, 7 de Outubro de 2006 15:24:44Assunto: RE: [obm-l] Res: [obm-l] Dúvida (Função e Divisibilidade)
Eu achei que eu ja tinha mostrado isso.Mas eu vou tentar fazer mais obvio.f(a+1) = f(a+2) + f(a)f(a+2) = f(a+3) + f(a+1)somando os dois ladosf(a+3) = - f(a)Ou seja, a cada 3 termos a funcao muda de sinalSe a quantidade de 3 termos (quantidade de mudancas de sinal) e impar a funcao acaba com sinal oposto, se nao acaba com o mesmo sinalse x = 3*n + a, entao f(x) = f(a) se n e par e f(x) = -f(a) se n e impar.Agora troca x por 2006 e a por 2.Melhorou?From: Jefferson Franca [EMAIL PROTECTED]Reply-To: obm-l@mat.puc-rio.brTo: obm-l@mat.puc-rio.brSubject: [obm-l] Res: [obm-l] Dúvida (Função e Divisibilidade)Date: Sat, 7 Oct 2006 09:56:32 -0700 (PDT)Será que não daria pra provar sua conjectura ? Dizer que f(x) = f(2) se x for par e que f(x) = - f(2) se x for ímpar é insuficiente, vc não
 acha?- Mensagem original De: Qwert Smith [EMAIL PROTECTED]Para: obm-l@mat.puc-rio.brEnviadas: Quinta-feira, 5 de Outubro de 2006 18:54:26Assunto: RE: [obm-l] Dúvida (Função e Divisibilidade)Vou tentar a primeira:f(3) = f(4) + f(2)f(4) = f(5) + f(3)somando os dois ladosf(5) = -f(2)Masf(6) = f(7) + f(5)f(7) = f(8) + f(6)e somando temosf(8)=-f(5)=f(2)logo se x = 3n + 2,f(x) = f(2) pra n par e f(x) = -f(2) pra n impar2006 = 3*n + 2 com n par, logo f(2006) = f(2) = 1 From: André Smaira [EMAIL PROTECTED] Reply-To: obm-l@mat.puc-rio.br To: obm-l@mat.puc-rio.br Subject: [obm-l] Dúvida (Função e Divisibilidade) Date: Thu, 5 Oct 2006
 14:04:43 -0300 (ART)  Apesar de acertar (foi meio na sorte), não consegui resolver estes dois exercícios da Olimpíada Mineira de Matemática. Se vcs souberem resolver me passem a resolucao:   5-) Considere uma função que tem a seguinte propriedade: f(x+1) + f(x-1) = f(x) com x inteiro. Se f(2) = 1, qual o valor de f(2006)?   a) -1  b) 0  c) 1  d) 28-) Sabendo que n é um número natural e que a divisão de n por 5 deixa resto 1; por 7 deixa resto 5 e por 9 também deixa resto 5, qual é o resto da divisão (n + 2)*(n + 1)^2 por 315?   a) 2  b) 5  c) 11  d) 25
   Agradeço antecipadamente,  André Smaira   - Novidade no Yahoo! Mail: receba alertas de novas mensagens no seu celular. Registre seu aparelho agora!=Instruções para entrar na lista, sair da lista e usar a lista emhttp://www.mat.puc-rio.br/~nicolau/olimp/obm-l.html=___Novidade no Yahoo! Mail: receba alertas de novas mensagens no seu celular. Registre seu aparelho agora!http://br.mobile.yahoo.com/mailalertas/=Instruções para entrar na lista, sair da lista e usar a lista emhttp://www.mat.puc-rio.br/~nicolau/olimp/obm-l.html=
		 
Novidade no Yahoo! Mail: receba alertas de novas mensagens no seu celular. Registre seu aparelho agora!

[obm-l] Res: [obm-l] Dúvida (Função e Divisibilidade )

2006-10-07 Por tôpico Jefferson Franca
Será que não daria pra provar sua conjectura ? Dizer que f(x) = f(2) se x for par e que f(x) = - f(2) se x for ímpar é insuficiente, vc não acha?
- Mensagem original De: Qwert Smith [EMAIL PROTECTED]Para: obm-l@mat.puc-rio.brEnviadas: Quinta-feira, 5 de Outubro de 2006 18:54:26Assunto: RE: [obm-l] Dúvida (Função e Divisibilidade)
Vou tentar a primeira:f(3) = f(4) + f(2)f(4) = f(5) + f(3)somando os dois ladosf(5) = -f(2)Masf(6) = f(7) + f(5)f(7) = f(8) + f(6)e somando temosf(8)=-f(5)=f(2)logo se x = 3n + 2,f(x) = f(2) pra n par e f(x) = -f(2) pra n impar2006 = 3*n + 2 com n par, logo f(2006) = f(2) = 1From: André Smaira [EMAIL PROTECTED]Reply-To: obm-l@mat.puc-rio.brTo: obm-l@mat.puc-rio.brSubject: [obm-l] Dúvida (Função e Divisibilidade)Date: Thu, 5 Oct 2006 14:04:43 -0300 (ART)Apesar de acertar (foi meio na sorte), não consegui resolver estes dois exercícios da Olimpíada Mineira de Matemática. Se vcs souberem resolver me passem a resolucao: 5-) Considere uma função que tem a seguinte propriedade: f(x+1) + f(x-1) = f(x) com x inteiro. Se f(2) = 1, qual o valor de
 f(2006)? a) -1 b) 0 c) 1 d) 2 8-) Sabendo que n é um número natural e que a divisão de n por 5 deixa resto 1; por 7 deixa resto 5 e por 9 também deixa resto 5, qual é o resto da divisão (n + 2)*(n + 1)^2 por 315? a) 2 b) 5 c) 11 d) 25 Agradeço antecipadamente, André Smaira-Novidade no Yahoo! Mail: receba alertas de novas mensagens no seu celular. Registre seu aparelho agora!=Instruções para entrar na lista, sair da lista e usar a lista emhttp://www.mat.puc-rio.br/~nicolau/olimp/obm-l.html=
		 
Yahoo! Search 
Música para ver e ouvir: You're Beautiful, do James Blunt

[obm-l] Combinatória

2006-10-07 Por tôpico Jefferson Franca

---Existem 13 torcedores brasileiros que querem assitir ao jogo Brasil X Argentina.
Dentre esses torcedores existem 5 que estào vestindo as camisas de seus times preferidos, um torce pro São Paulo, outro para a Portuguesa, outro para o Palmeiras, outro para o Santos e outro para Corinthians.
De quantas maneiras podemos colocar esses torcedores na fila para a compra de ingressos se para evitar confrontos entre eles, nenhum torcedor de time paulista pode ficar do lado de outro torcedor paulista, ou seja, um torcedor que está usando a camisa de um time paulista não pode ter imediatamente na sua frente um torcedor de time paulista e nem imediatamente atrás dele, isto é, deve existir no mínimo um torcedor com a camisa da seleção entre dois torcedores que vestem camisas de times paulistas.?
		 
Novidade no Yahoo! Mail: receba alertas de novas mensagens no seu celular. Registre seu aparelho agora!

[obm-l] Res: [obm-l]Fw: Olímpica de PA

2006-10-07 Por tôpico Jefferson Franca
Oi Gustavo.
Eu fiz o seguinte: Escreva a2 + a4 + a6 +...+ a(n) = 161 (I) e a1 + a3+ a5++ a(n - 1) = 140 ( II), agora some ( I) com ( II), e note que 301 = (a1 + a(n))n/2 e que a soma dos termos equidistantes dos extremos é exatamente igual a a1 + a(n) , ou seja, 301 = 43.n/2, o que dá n = 14.
Abs
- Mensagem original De: Gustavo Duarte [EMAIL PROTECTED]Para: obm-l@mat.puc-rio.brEnviadas: Sábado, 7 de Outubro de 2006 8:36:16Assunto: [obm-l]Fw: Olímpica de PA



Agreadeço desde ja se alguém puder alguma ajuda  

Em uma certa PA a soma dos termos de ordem IMPAR é 140, a soma dos termos de ordem PAR é 161,e a soma de dois termos equidistantes dos extremos é 43. Qual o numero de termos desa PA ? SOL. 14

		 
Novidade no Yahoo! Mail: receba alertas de novas mensagens no seu celular. Registre seu aparelho agora!

Re: [obm-l] trigonometria

2006-09-28 Por tôpico Jefferson Franca
Vejo duas soluções, a primeira seria escrever tangente como seno sobre cosseno, ou seja, tg(pi/7) = (sen(pi/7))/ cos(pi/7) e assim por diante, agora multiplique em cima e embaixo por 2sen(pi/14), e transforme o produto em somas.  Infelizmente, estou extremamente atrasado, depois te dou a solução com mais detalhes. Espero ter ajudado um pouco.  Abs[EMAIL PROTECTED] escreveu:  Quanto vale (tg pi /7).(tg 2pi /7).(tg 3pi /7)?=Instruções para entrar na lista, sair da lista e usar a lista emhttp://www.mat.puc-rio.br/~nicolau/olimp/obm-l.html= 
		 
Você quer respostas para suas perguntas? Ou você sabe muito e quer compartilhar seu conhecimento? Experimente o Yahoo! Respostas!

Re: [obm-l] OBM Prova

2006-09-01 Por tôpico Jefferson Franca
Boa sorte para vc tbm.Saulo [EMAIL PROTECTED] escreveu:  Saudações á todos do fórum!Queria desejar uma ótima prova amanhã para quem for fazer tanto do nível médio quanto o Universitário. Essa será a primeira vez que participo oficialmente dessa prova já que anteriormente tinha participado mas como convidado pela escola onde estudava que era o Elite, pois não era aluno do E.M. e então participei pq sempre tive vontade de participar e nunca tive oportunidade nas outras escolas.Hoje estou cursando Engenharia Mecânica e Matemática, 5° e 1° respectivamente e estou muito feliz por está participando, me preparei com algumas revista da Eureka em que meu professor Marcos do CEFET me deu, e livros de Cálculos e de Matemática que ganhei de amigos, e também por aqui.Gostaria de agradecer aos amigos que sempre
 enviavam questões, problemas e curiosidades sempre tentei buscar forças para resolver, ou então aguardar respostas dos amigos para ver se as minhas estavam pelo menos iguais. Realmente aqui na lista tem muitos feras, e por isso ficava meio que com receio de enviar questões e também enviar minhas soluções, mas isso já estou conseguindo vencer.Bem só não estou 100% feliz porque o único do CEFET q irá participar do Nível Superior será eu! Já que outros amigos desistiram de última hora de participar da prova alegando que essa prova não sirva para nada, confesso que fiquei entristecido com tais atitudes. Mas carrego minha cruz e sigo meu caminho,e espero realizar uma boa prova, como os demais amigos também.Um abraço, Boa Prova para todos, Deus nos
 abençoe.Saulo.--=Instruções para entrar na lista, sair da lista e usar a lista emhttp://www.mat.puc-rio.br/~nicolau/olimp/obm-l.html= 
		 
Yahoo! Search 
Música para ver e ouvir: You're Beautiful, do James Blunt

Re: [obm-l] resto da divisao

2006-08-11 Por tôpico Jefferson Franca
Acho que pra primeira questão vc poderia usar o algoritmo da divisão de Euclides, ou seja, escrever que p(x) = x^81+x^40-3x^25+x^8+x quando dividido por d(x) =x^3-x^2 tem quociente q(x) e resto r(x) que deve ser um polinômio de grau menor que 3 e maior que zero.  Note que podemos fatorar d(x), isto é, d(x) = x^2 (x - 1), assim concluímos que as raízes de d(x) são 0 e 1 e esrever r(x) como a*x^2 + b*x + c, logo  p(x) = d(x) q(x) + r(x), fazendo x assumir os valores 0 e 1, iremos encontrar as constantes a, b e c, ou seja, o polinômio r(x).  A segunda é mais fácil, faça p(1), depois p(1) e pronto.ilhadepaqueta [EMAIL PROTECTED] escreveu:  Por favor,1) Qual o resto da divisão de x^81+x^40-3x^25+x^8+xpor
 x^3-x^2 ?2) O polinômio P(x)=x^5-5x^4-x^3+mx^2+nx+pé divisível por (x^2-1)(x-1), quando:a) m = n + pb) m - p + n = 13c) m + p = nd) m + n = pe) n + p = 2mObrigado mais uma vez. 
		 
Yahoo! Search 
Música para ver e ouvir: You're Beautiful, do James Blunt

Re: [obm-l] Alg. Linear

2006-08-02 Por tôpico Jefferson Franca
Tenho quase certeza que essa é uma questão que caiu no vestiba do ime ou que está num livro de geometria do carroneth.  Klaus Ferraz [EMAIL PROTECTED] escreveu:  Mostre que todo ponto pertencente ao circulo circunscrito a um triangulo é foco de uma parabola tangente aos tres lados do triangulo.  Yahoo! Acesso Grátis - Internet rápida e grátis. Instale o discador agora!  
		 
O Yahoo! está de cara nova. Venha conferir!

Re: [obm-l] Trigonometria

2006-03-12 Por tôpico Jefferson Franca
Acho que uma boa idéia seria usar expansão de Taylor.Adélman de Barros Villa Neto [EMAIL PROTECTED] escreveu:  Qual o valor aproximado de sen 6 + tg 3 ?=Instruções para entrar na lista, sair da lista e usar a lista emhttp://www.mat.puc-rio.br/~nicolau/olimp/obm-l.html=
		 
Yahoo! Search 
Dê uma espiadinha e saiba tudo sobre o Big Brother Brasil.

Re: [obm-l] geometria

2006-03-04 Por tôpico Jefferson Franca
Boa noite, Anna.  Inicialmente seria bom fazer o desenho para ter mais clareza da solução.  A única posição para o quadrado é aquela onde seu centro coincide com o centro do semicírculo.  Unindo o centro até um dos vértices teremos um triângulo retângulo cujos catetos são l( lado do quadrado) e l/2 e a hipotenusa é R(raio do semicírculo), basta usar o bom e velho teorema do Pit., achando a relação R^2 =(5*l^2)/4, porém, como a área do quadrado é 36 cm^2, tem-se que R = 3*(5)^1/2.  Se vc usar a idéia de média geométrica no item b, vai descobrir que a diagonal do quadrado mede 16*(2)^1/2Anna Luisa [EMAIL PROTECTED] escreveu:  Quem tiver um tempo, me ajuda por favor, pq minha res!
posta não
 bate c/ o gabarito.1) Tendo um quadrado inscrito num semicírculo de raio re sabendo que a área do quadrado e 36cm2, podemos afirmar que o valor de r é:2) As medidas do lado, do perímetro e da área de um quadrado estão, nesta ordem, em progressão geométrica. A diagonal deste quadrado mede:Obrigada.  Anna.
		 
Yahoo! Search 
Dê uma espiadinha e saiba tudo sobre o Big Brother Brasil.

Re: [obm-l] Fw: congruência

2006-03-04 Por tôpico Jefferson Franca
Vc já tentou fazer o seguinte: Chame 10^k de y e resolva a seguinte equação diofantina y - 23x = 8, acho que sai por aí.Leo [EMAIL PROTECTED] escreveu:- Original Message -   From: Leo   To: obm-l@mat.puc-rio.br   Sent: Saturday, March 04, 2006 12:16 AM  Subject: Fw: congruência  - Original Message -   From: Leo   To: obm-l@mat.puc-rio.br   Sent: Friday, March 03, 2006 8:11 PM  Subject: congruênciaComo resolver a seguinte congruência  10^k cong 8 (mod 23) ... pra k=2 eh verdadeira mas como achar o caso geral???
		 
Yahoo! Acesso Grátis 
Internet rápida e grátis. Instale o discador agora!

Re: [obm-l] trigonometria

2005-11-17 Por tôpico Jefferson Franca
Fazendo (senx)^6 + (cosx)^6 = ((senx)^2)^3 +((cosx)^2)^3 e usando as identidadesa^3 +b^3 =(a+b) *(a^2 - ab + b^2)e depois a^2 + b^2 = (a+b)^2 - 2ab , chega-se que y = 1 - (3/4)*(sen2x)^2, observe que o valor mínimo de (sen2x)^2 é zero e, para este valor tem-se que y = 1, enquanto que o valor máximo de (sen2x)^2 é um e, para este valor tem-se y = 1/4Marcio Cohen [EMAIL PROTECTED] escre!
veu:  
 Pondo(senx)^2 =1/2+t, (cosx)^2=1/2-t, -1/2=t=1/2   y = (1/2+t)^3 + (1/2-t)^3 = 1/4 + 3t^2 tem mínimo em t=0 (y=1/4) e máximo em t=+-1/2 (y=1)  Observe que y=1/4 para x=pi/4, 3pi/4, 5pi/4, ... logo o período é maior ou igual a pi/2.  Por outro lado, trocar x por x+pi/2 não muda o valor de y, logo o período é exatamente pi/2.- Original Message -   From: Guilherme Neves   To: obm-l@mat.puc-rio.br   Sent: Wednesday, November 16, 2005 9:46 PM  Subject: [obm-l] trigonometria  Achar os valores maximo e minimo e o periodo da função y=(sen(x))^6 + (cos(x))^6   fiz de uma maneira mt trabalhosa que se ninguem tiver feito igualmente eu coloco aqui. = Instruções para entrar na lista, sair da lista e usar a lista em http://www.mat.puc-rio.br/~nicolau/olimp/obm-l.html =
 
		 
Yahoo! Acesso Grátis: Internet rápida e grátis.Instale o discador agora!

Re: [obm-l] Algebra inteiros

2005-11-17 Por tôpico Jefferson Franca
Suponha que m é da forma 2k + 1, com k inteiro, logo m^2 - 1 = (2k + 1) ^2 - 1 = 4*k^2 + 4*k + 1- 1 = 4k(K + 1), ou seja, 4 vezes dois inteiros consecutivos , isto é, múltiplo de 8.  Suponha agora que m = 2k - 1, com k inteiro, logo m^2 - 1 = (2k - 1)^2 - 1 = 4k^2 - 4k + 1 - 1 = 4k(k - 1), novamente temos quatro vezes dois inteiros consecutivos, isto é, um múltiplo de 8.  CQD[EMAIL PROTECTED] escreveu:   On Mie Nov 16 23:17 , marcio aparecido <[EMAIL PROTECTED]>sent:Seja m um número ímpar, prove que m^2 -1 é divisil por 8.=Instruções para entrar na lista, sair da lista e usar a lista emhttp://www.mat.puc-rio.br/~nicolau/olimp/obm-l.html= FLAGS (\Seen))m = 2k+1 = m^2 - 1 = (4k^2 + 4k + 1) - 1 = 4k(k + 1)* Se k é par, 4k é divisível por 8.* Se k é ímpar, k + 1 é par, e 4(k + 1) é divisível por 8.[]s,Márcio.=Instruções para entrar na lista, sair da lista e usar a lista
 emhttp://www.mat.puc-rio.br/~nicolau/olimp/obm-l.html=  
		 
Yahoo! Acesso Grátis: Internet rápida e grátis.Instale o discador agora!

Re: [obm-l] Re: [obm-l] combinatória

2005-11-06 Por tôpico Jefferson Franca








Valeu CarlosCarlos Gomes [EMAIL PROTECTED] escreveu: 




Jeffferson, é o seguinte:

 Calcula-se todas as combinações dos 10 pontos 2 a 2[ C(10,2)=45 ]e retira-se as combinações dos 4 pontos que estão alinhados, isto é C(4,2)=6 o que geraria 45-6=39. Mas ao retirar todas as combinações dos 4 pontos alinhados tomados 2 a 2 retiramos também a reta na qual eles se situam, assim para obter a quantidade correta de retas precisamos "devolver" esta reta o que faz com que a resposta correata seja 45-6+1=40.

Valew, Cgomes

- Original Message ----- 
From: Jefferson Franca 
To: obm-l@mat.puc-rio.br 
Sent: Sunday, November 06, 2005 2:00 AM
Subject: [obm-l] combinatória





Será que alguém poderia fazer o favor de liquidar com essa questão que atualmente é motivo de insônia para mim? A questão é a seguinte : Considere, num plano, 10 pontos distintos entre si. Suponha que 4 desses pontos pertençam a uma mesma reta e que 2 quaisquer dos demais não estejam alinhados com nenhum pontos restantes. Calcule o número de retas determinadas pelos 10 pontos 





Yahoo! Acesso Grátis: Internet rápida e grátis.Instale o discador agora!-- Esta mensagem foi verificada pelo sistema de anti-virus e acredita-se estar livre de perigo. 



Internal Virus Database is out-of-date.Checked by AVG Anti-Virus.Version: 7.0.344 / Virus Database: 267.11.14/128 - Release Date: 10/10/2005-- Esta mensagem foi verificada pelo sistema de anti-virus e acredita-se estar livre de perigo. 
		 
Yahoo! Acesso Grátis: Internet rápida e grátis.Instale o discador agora!

Re: [obm-l] trigo (essa eh f**!)

2005-11-06 Por tôpico Jefferson Franca








Acho que vc pode fazer o seguinte: Chame o primeiro membro de Y , por exemplo, e depois multiplique tudo por 2sen(r/2), lembre de que a_2 = a_1 + r, a_3= a_1 + 2r,, Rodrigo Augusto [EMAIL PROTECTED] escreveu: 
prove a identidade abaixo, sabendo que os arcos estao em pa de razao r:cos(a_1) + cos(a_2) + cos(a_3) +... + cos(a_n) = {cos[(a_1/2 + a_n/2)]*sen(nr/2)}/sen(r/2)valeu e bom domingo pra vcs!_MSN Messenger: converse online com seus amigos . http://messenger.msn.com.br=Instruções para entrar na lista, sair da lista e usar a lista emhttp://www.mat.puc-rio.br/~nicolau/olimp/obm-l.html=
		 
Yahoo! Acesso Grátis: Internet rápida e grátis.Instale o discador agora!

Re: [obm-l] trigo (essa eh f**!)

2005-11-06 Por tôpico Jefferson Franca








O objetivo é transformar a soma do primeiro membro numa soma telescópica.Rodrigo Augusto [EMAIL PROTECTED] escreveu: 
prove a identidade abaixo, sabendo que os arcos estao em pa de razao r:cos(a_1) + cos(a_2) + cos(a_3) +... + cos(a_n) = {cos[(a_1/2 + a_n/2)]*sen(nr/2)}/sen(r/2)valeu e bom domingo pra vcs!_MSN Messenger: converse online com seus amigos . http://messenger.msn.com.br=Instruções para entrar na lista, sair da lista e usar a lista emhttp://www.mat.puc-rio.br/~nicolau/olimp/obm-l.html=
		 
Yahoo! Acesso Grátis: Internet rápida e grátis.Instale o discador agora!

Re: [obm-l] Limite superior para a soma de logaritmos

2005-11-06 Por tôpico Jefferson Franca








Bom dia , para todos da lista. Fabinho de onde vc tirou essa de logk + log(n - k)  (logn)/2 ? Que bruxaria é essa?Fábio Dias Moreira [EMAIL PROTECTED] escreveu: 
On Sun, 2005-11-06 at 00:07 -0200, Denisson wrote: Em 06/11/05, Fábio Dias Moreira <[EMAIL PROTECTED]>escreveu:  On Sat, 2005-11-05 at 23:37 -0200, Denisson wrote:   Boa noite pessoal,  Tou tentando encontrar uma função que limite superiormente a soma   log1 + log2 + ... + log(n)... e não tou conseguindo...   Alguma ajuda?   log 1 + log 2 + ... + log n = log n + log n + ... + log n = n * log  n.   Tem razão, agora essa função também é limite inferior?Mais ou menos... Não é muito difícil ver quelog k + log (n-k)  (log n) / 2para todo n = 3 e 0  k  n. Logolog 1 + log 2 + ... + log n  (n * log n) / 4para todo n suficientemente
 grande.[]s, []s,  -- Fábio Dias Moreira   = Instruções para entrar na lista, sair da lista e usar a lista em http://www.mat.puc-rio.br/~nicolau/olimp/obm-l.html =--  Denisson  "Os homens esqueceram desta verdade; mas tu não a deves esquecer:  É só com o coração que se pode ver direito. O essencial é invisível aos olhos!" (Saint Exupèrry)-- Fábio Dias Moreira=Instruções para entrar na lista, sair da lista e usar a lista
 emhttp://www.mat.puc-rio.br/~nicolau/olimp/obm-l.html=
		 
Yahoo! Acesso Grátis: Internet rápida e grátis.Instale o discador agora!

[obm-l] combinatória

2005-11-05 Por tôpico Jefferson Franca




Será que alguém poderia fazer o favor de liquidar com essa questão que atualmente é motivo de insônia para mim? A questão é a seguinte : Considere, num plano, 10 pontos distintos entre si. Suponha que 4 desses pontos pertençam a uma mesma reta e que 2 quaisquer dos demais não estejam alinhados com nenhum pontos restantes. Calcule o número de retas determinadas pelos 10 pontos 



		 
Yahoo! Acesso Grátis: Internet rápida e grátis.Instale o discador agora!

Re: [obm-l] equação

2005-11-04 Por tôpico Jefferson Franca
Que tal se vc fizer assim: x^2 + x^(1/2)= 16 + 2 -- x^2 - 16 + sqrt(x) - 2 = 0 --- (x + 4)(x - 4) + sqrt(x) - 2 = 0 -- (x+4)(sqrt(x)+2)(sqrt(x)-2) + sqrt(x) - 2 = 0 -- (sqrt(x) - 2) ((sqrt(x) + 2)(x + 4) + 1) = 0, perceba que se considerarmos 
x real e positivo, entâo a única maneira de anular esse produto é se sqrt(x) - 2 = 0, ou seja, x = 4Aldo Munhoz [EMAIL PROTECTED] escreveu:
As raízes são:x=4x=-4/3-1/6*[1+sqrt(3)i]*[1339/2-9sqrt(457)/2]^(1/3)-1/6*[1-sqrt(3)i]*[1339/2+9sqrt(457)/2]^(1/3)x=-4/3-1/6*[1-sqrt(3)i]*[1339/2-9sqrt(457)/2]^(1/3)-1/6*[1+sqrt(3)i]*[1339/2+9sqrt(457)/2]^(1/3)As duas últimas, ao serem simplificadas, mostram-se puramente complexas, portanto, a única solução real é x=4.Abraços,AldoIuri wrote: 
Tentei aqui e encontrei apenas 4 como raiz. Abaixando o grau, fica y^3 + 2y^2 + 4y +9 = 0, onde y=sqrt(x). Pelo menos mais uma das raizes deve ser real, mas nao a encontrei. É alguma coisa irracional.
Em 02/11/05, Marcelo de Oliveira Andrade [EMAIL PROTECTED] escreveu: 
eu sei que em pleno feirado é f**...hehe... mas se alguem estiver aí equiser me ajudar com esse exercício! x^2 + sqrt(x)-18 = 0, x=0.muito obrigado_Chegou o que faltava: MSN Acesso Grátis. Instale Já!http://www.msn.com.br/discador=Instruções para entrar na lista, sair da lista e usar a lista emhttp://www.mat.puc-rio.br/~nicolau/olimp/obm-l.html== Instruções para entrar !
na lista,
 sair da lista e usar a lista em http://www.mat.puc-rio.br/~nicolau/olimp/obm-l.html = 
		 
Yahoo! Acesso Grátis: Internet rápida e grátis.Instale o discador agora!

Re: [obm-l] Ajuda - Proporção

2005-10-01 Por tôpico Jefferson Franca








Supondo que k seja a constante de proporcionalidade, então 70 pode ser divididoem três partes a, b e c , tais que a = 2k, b = 3k e c = 5k, logo a menor parte é a e a maior parte é c.
Como a + b + c = 70, então 2k + 3k+ 5k = 70, ou seja, k = 7, logo a =14 e c =35admath [EMAIL PROTECTED] escreveu: 


Olá

Já li diversas teorias sobre proporcionalidade só que não consigo entender estes dois problemas de maneira alguma. Alguém pode me explicar de uma maneira bem didática?

1) Dividindo 70 em partes proporcionais a 2, 3 e 5, a soma entre a menor e a maior parte é quanto?

2) A proporção entre as medalhas de ouro, prata e bronze de um atletaé 3 : 4 : 7, respectivamente. Quantas medalhas de ouro, prata e bronze espera-se que esse atleta obtenha em 70 jogos, se essa proporção se mantiver e ele conquistar medalhas em todos os jogos?

-Posso falar que o método da regra de 3 éo mesmo processoquando lidamos com grandezas proporcionais?

obrigado.
		 
Novo Yahoo! Messenger com voz: ligações, Yahoo! Avatars, novos emoticons e muito mais. Instale agora!

[obm-l] Livros da MIR

2005-09-04 Por tôpico Jefferson Franca




Aos amigos da lista que estão interessados em adquirir livros da extinta editora mir, um dos endereços é www.mir.ru outro é 
www.urss.ru . Certamente , um dos meus melhores livros de cálculo foi o demidovitch , eu o indico a todos que querem uma boa preparação nesse ramo da matemática.
Abraços



		 
Yahoo! Messenger com voz: PROMOÇÃO VOCÊ PODE LEVAR UMA VIAGEM NA CONVERSA. Participe!

Re: [obm-l] Livros da MIR

2005-09-04 Por tôpico Jefferson Franca

Mesmo assim vc ajudou muito dando esse endereço.
ValeuMarcio [EMAIL PROTECTED] escreveu:
De fato, passei o endereço errado. O correto (testei antes de escrever) é realmente www.livifusp.com.br.[]s,Márcio.On Sun, 04 Sep 2005 14:06:12 -0700, Bernardo Freitas Paulo da Costa <[EMAIL PROTECTED]>wrote: Cara, o site mudou de nome (tinha uma pagina velha que ainda tinha isso, a unica no Google, e que disse que isso era a Livraria de Fisica da Usp), agora tem um "i" a mais: http://www.livifusp.com.br/. Espero que seja este mesmo. Abraços,-- Using Opera's revolutionary e-mail client: http://www.opera.com/mail/=Instruções para entrar na lista, sair da lista e usar a lista
 emhttp://www.mat.puc-rio.br/~nicolau/olimp/obm-l.html=
		 
Yahoo! Messenger com voz: PROMOÇÃO VOCÊ PODE LEVAR UMA VIAGEM NA CONVERSA. Participe!

Re: [obm-l] cos(2*pi/17)

2005-08-25 Por tôpico Jefferson Franca
Mais uma vez , eu agradeço a ajuda e agora acho que posso dormir em paz, mesmo depois do árduo trabalho que terei para resolver esta questão.
Valeu"Nicolau C. Saldanha" [EMAIL PROTECTED] escreveu:
On Thu, Aug 25, 2005 at 01:19:04AM -0300, Jefferson Franca wrote: Nem vou perguntar de onde tirou essa idéia, mas valeu pela solução.Em um curso de álgebra que cubra teoria de Galois este tipo de coisaé explicada com mais contexto. Eu dei um esboço rápido e elementar.Aliás, todos estes "prove que" são braçais. Para isso, tomea = cos(2*pi/17) + i sen(2*pi/17) de tal forma quexk = a^k + a^(17-k). Expanda tudo em termos de a, use o fatoque 1+a+a^2+...+a^16 = 0, faça muitas contas com polinômioscom coeficientes inteiros e tudo segue.[]s, N.=Instruções para entrar na lista, sair da lista e usar a lista
 emhttp://www.mat.puc-rio.br/~nicolau/olimp/obm-l.html=__Converse com seus amigos em tempo real com o Yahoo! Messenger http://br.download.yahoo.com/messenger/ 

Re: [obm-l] cos(2*pi/17)

2005-08-24 Por tôpico Jefferson Franca
Nem vou perguntar de onde tirou essa idéia, mas valeu pela solução."Nicolau C. Saldanha" [EMAIL PROTECTED] escreveu:
On Mon, Aug 22, 2005 at 01:26:34PM -0300, Jefferson Franca wrote: Como posso calcular o cosseno de 2pi/17 ?Vou dar só um esboço e a resposta.Defina x1, x2, ..., x8 como xk = 2*cos(2*k*pi/17).Prove que w1=x1+x2+x4+x8 e w2=x3+x5+x6+x7 são as raízes de w^2+w-4,donde w1 = (-1+sqrt(17))/2, w2 = (-1-sqrt(17))/2.Sejam z1=x1+x4 e z2=x2+x8. Prove que z1*z2 = -1. Como z1+z2=w1, conclua quez1 = (-1+sqrt(17)+sqrt(34-2*sqrt(17)))/4,z2 = (-1+sqrt(17)-sqrt(34-2*sqrt(17)))/4.Sejam y1=x1*x4, y2=x2*x8, y3=x3*x5, y4=x6*x7. Prove que y1*y2=-1.Sejam u1=y1+y2, u2=y3+y4.Prove que u1 e u2 são raízes de u^2+u-4 donde u1=w2, u2=w1.Conclua que y1 e y2 são raízes de y^2 - u1*y - 1, dondey1 = (-1-sqrt(17)+sqrt(34+2*sqrt(17)))/4,y2 = (-1-sqrt(17)-sqrt(34+2*sqrt(17)))/4.Como conhecemos os valores de z1=x1+x4 e y1=x1*x4,!
 fica
 fácil obter x1.[]s, N.=Instruções para entrar na lista, sair da lista e usar a lista emhttp://www.mat.puc-rio.br/~nicolau/olimp/obm-l.html=
		Yahoo! Acesso Grátis: Internet rápida e grátis. Instale o discador agora!

Re: [obm-l] videos impa

2005-08-22 Por tôpico Jefferson Franca


Como posso calcular o cosseno de 2pi/17 ?
Pensei em usar a técnica tradicional de construir um polígono regular de 17 lados, mas ..

		Yahoo! Acesso Grátis: Internet rápida e grátis. Instale o discador agora!

Re: [obm-l] trigonometria

2005-08-18 Por tôpico Jefferson Franca
Sei lá, isso é muito subjetivo, acho que daria 3.Johann Peter Gustav Lejeune Dirichlet [EMAIL PROTECTED] escreveu:
Voce talvez nao tenha entendido a minha colocacao...A trigonometria acaba ao se descobrir que o produto eigual a soma. Depois disso, e Teoria dos numeros.Pondo de outra forma: quantos pontos, de no maximo 10,voce daria para quem nao resolveu a parte do x+y+z=xyzmas chegou nas tangentes? Eu nao daria mais que 4.5--- Jefferson Franca <[EMAIL PROTECTED]>escreveu: Obrigado , mas acho que vc não analisou direito esta questão e perceba que ela é sim de trigonometria!!! Valeu e um abraço  Johann Peter Gustav Lejeune Dirichlet <[EMAIL PROTECTED]>escreveu: Ue, pelo menos um dos caras nao e maior que 2 ( o caso do 1 ai escrito, e do 2, que nao e maior ue 2 pois e igual...). E alias vamos fazer logo isso antes que nao de mais! !
; Temos
 1/xy+1/xz+1/yz=1. Se xy=c, xz=b, yz=a, temos 1/a+1/b+1/c=1 Suponha a=b=c. Entao se c=4, temos 1/a+1/b+1/c=3/41, absurdo! Logo c=3 Ai e so testar!  *c=3 1/a+1/b=2/3 Se b=4 entao 1/a+1/b = 2/4 = 1/2  2/3, nao da! Entao b=3 Mas 3=c=b=3, o que da b=c=3 E (a,b,c)=(3,3,3) da certo.  *c=2 Ai 1/a+1/b=1/2 E entao b=4  Testa de novo! **b=4 1/a=1/4 e ai a=4, (a,b,c)=(4,4,2)  **b=3 1/a=1/6 (a,b,c)=(6,3,2)  **b=2 1/a=0. Nao da!  c=1, nao serve!  Ai e so transformar cada a,b,c em x,y,z:  (xy,xz,yz)=(3,3,3) (xyz)^2=27 (xy,xz,yz)=(4,4,2) (xyz)^2=32 (xy,xz,yz)=(6,3,2) (xyz)^2=36=6^2A primeira nao serve (3 nao e quadrado perfeito)!
.
 A segunda tambem nao...  Ja temos entao xyz=6, e agora sem dificuldade comclui-se que a solucao apresentada anteriormente e unica (alias e exatamente a solucao que voce achou e satisfaz uma porrada de requisitos adicionais...)   P.S.: Como eu ja desconfiava este problema nao tem nada de trigonometria!  --- Jefferson Franca  escreveu:   E se tivermos, por exemplo, X = 3, Y= 2 e Z = 1 ?Johann Peter Gustav Lejeune Dirichlet   escreveu:tg C= tg  A + tg B/ 1- tg A. tgB  tg C - tg A . tg B .tg C = tg A+tg B  tg A . tg B .tg C = tg A+ tg B + tg COu seja, se acharmos X,Y,Z tais que   XYZ=X+Y+Z, o problema acaba.Isto e algo facil de resolver, e prova que a solucao  e  mesmo unica.!
 
 Como o Caio ja disse, e facil ver que pelo menos um  dos caras X,Y,Z e no maximo 2.--- Jefferson Franca   escreveu: Desculpe, Caio,mas desconfio que não seja.  [EMAIL PROTECTED] escreveu:   Suponha que 2 dessas tangentes sejam maiores que  2,   ou seja,   tg A = 2 + x   tg B = 2 + y (x,y 0)  A + B + C = 180   A + B = 180 -Ctg (A + B ) = - tg C   tg A + tg B/ 1- tg A. tgB = (4+x+y)/1- (4 + 2(x+y)   +xy)   = (4 + x + y )/ -(3 + 2(x+y) +xy)  tg C = (4+x+y)/(3+ 2x + 2y +xy)  Teremos que tg C  2 =   4 + x + y  6 + 4x + 4y + 2xy   !
t;= 2
 + 3x + 3y + 2xy0Como por hipótese, x e y sao positivos , essa soma   nunca é negativa   ou seja, nunca vale que tg C  2  ou seja, é impossível ter as 3 tangentes maior que  2   (simultaneamente)Ou seja, a solução dada pelo nosso amigo é unica!  ''-- Mensagem Original --   ''Date: Wed, 10 Aug 2005 12:19:32 -0700   ''From: Marcio   ''To: obm-l@mat.puc-rio.br   ''Subject: Re: [obm-l] trigonometria   ''Reply-To: obm-l@mat.puc-rio.br   ''   ''   ''On Wed, 10 Aug 2005 0!
6:20:13
 -0700, Jefferson   Franca'' wrote:   ''   '' Será que alguém já viu esta questão ou tem   alguma idéia de como resolver   '''' ?   '' Sejam a ,b e c ângulos internos de  umtriângulo   e, supondo que as   '' tangentes dos três ângulos sejam números   inteiros e positivos, calcule   '''' essas tangentes.   '' Valeu   ''   ''   __   '' Converse com seus amigos em tempo real com o   Yahoo! Messenger   '' http://br.download.yahoo.com/messenger/  
 ''   ''   ''   ''--''Using Opera's revolutionary e-mail client:   http://www.opera.com/mail/   ''   ''Oi, Jefferson.   ''   ''Se não errei nada, aqui vai.   ''   ''Ângulos: a, b e c   ''   ''a + b + c = 180 = tg(a + b + c)= tg 180, ou   seja, tg(a + b + c) = 0   ''   ''Daí, tg(a + b) + tg(c) = 0.   ''   ''No final das contas, chega-se a   ''   ''tg a + tg b + tg c = (tg a)(tg b)(tg c)   ''   ''Como as tangentes são números inteiros e   positivos, uma opção (não sei   se   ''  !
; 
 ''única) é   ''   ''tg a = 1, tg b = 2 e tg c = 3   ''   ''   ''[]s,   ''   ''Márcio.   ''= === message truncated ===___ Yahoo! Acesso Grátis - Internet rápida e grátis. Instale o discador agora! http://br.acesso.yahoo.com/=Instruções para entrar na lista, sair da lista e usar a lista emhttp://www.mat.puc-rio.br/~nicolau/olimp/obm-l.html=
		Yahoo! Acesso Grátis: Internet rápida e grátis. Instale o discador agora!

Re: [obm-l] trigonometria

2005-08-18 Por tôpico Jefferson Franca
Agora foi vc quem não entendeu.
Já tentei resolver esta questão e , acho que não tem outro meio de analisar a questão a não ser "conjecturar"as soluções e a partir delas, ir encontrando as outras.
Valeu!Johann Peter Gustav Lejeune Dirichlet [EMAIL PROTECTED] escreveu:
Ao inves de conjecturar voce devia partir pra porradacom a expressao.bc=2,ac=4 e ab=4 e a unica maneira possivel de estabagaça ter alguma solucao.Se os tres valores acima fossem 3, o produtodeles(quadrado perfeito) seria 27(nao quadradoperfeito).Alias, aonde esta a falha? Do que exsatamente tu estasreclamando?--- Jefferson Franca <[EMAIL PROTECTED]>escreveu: Meu caro amigo e ex aluno Eurico, vc não deve ter pensado no seguinte: a + b + c = a.b.c = (1/bc) + (1/ac) + (1/ab) = 1  e, suponha que bc = 3, ac = 3 e ab = 3, e se tivéssemos bc = 2, ac = 4 e ab = 4? Eu, sinceramente acho que esta questão não tem só uma solução como vc afirma.  Valeu e abraço   Antonio Eurico Dias <[EMAIL PROTECTED]>escreveu:  E completando o
 raciocinio do Dirichlet:  TgA + TgB + TgC = TgA. TgB. TgC  a + b + c = a.b.c = abc - a = b + c = a (bc - 1) = b + c  a = (b + c) / (bc - 1), como a é inteiro positivo: b + c = bc - 1 = bc - b = c + 1  b.(c - 1) = c + 1 = b = (c+1)/(c-1) = b = 1 + 2/(c-1)  Logo, como b também é inteiro c - 1 = 2 ou c -1 = 1, que levam ao unico conjunto de solucoes inteiras positivas   (1 , 2 , 3)...Eurico  Sistema Elite de Ensino - Unidade Belém - Start your day with Yahoo! - make it your home page  __ Converse com seus amigos em tempo real com o Yahoo! Messenger 
 http://br.download.yahoo.com/messenger/ ___ Yahoo! Acesso Grátis - Internet rápida e grátis. Instale o discador agora! http://br.acesso.yahoo.com/=Instruções para entrar na lista, sair da lista e usar a lista emhttp://www.mat.puc-rio.br/~nicolau/olimp/obm-l.html=
		Yahoo! Acesso Grátis: Internet rápida e grátis. Instale o discador agora!

Re: [obm-l] trigonometria

2005-08-18 Por tôpico Jefferson Franca
Eurico, a questão exige soluções e dar resposta incompleta não é bom.
E , ela não é questão de teoria dos números , aliás, tem um pouco de aritmética e um pouco de trigonometria,mesmo assim se faz necessária a solução completa.
ValeuAntonio Eurico Dias [EMAIL PROTECTED] escreveu:
Essa questao é identica a 12a. da prova da Unicamp de2003... Como o objetivo é encontrar valores inteiros epositivos para as tangentes não há motivo para tantosdevaneios... É pura teoria dos números...Eurico DiasStart your day with Yahoo! - make it your home page http://www.yahoo.com/r/hs =Instruções para entrar na lista, sair da lista e usar a lista emhttp://www.mat.puc-rio.br/~nicolau/olimp/obm-l.html=
		Yahoo! Acesso Grátis: Internet rápida e grátis. Instale o discador agora!

Re: [obm-l] trigonometria

2005-08-18 Por tôpico Jefferson Franca
Quem disse que as tangentes são números inteiros ,positivos e CONSECUTIVOS?
[EMAIL PROTECTED] escreveu:
Jefferson, eu acho que vc nao está aceitando pelo fato de que nao encontramosuma equação que nos dá todas as soluções e portanto prova que a soluçãoé unica.nao seja por isso:Bom, se A, B, C sao os angulos internos de um triangulo entaotgA+tgB+tgC = tgA.tgB.tgCSeja tgB= x, entao tgA=x-1 tgC=x+1 (x inteiro e positivo)(x+1)+x+(x-1)=(x+1).x.(x-1)3x = x(x²-1)3x = x³- xx³ = 4xComo x0 pois x-1 deve ser positivo, entao x²=4como x deve ser positivo, a unica solução aceita pra x = 2entao as tangentes sao 1, 2, 3''-- Mensagem Original --''Date: Wed, 17 Aug 2005 05:47:33 -0700 (PDT)''From: Antonio Eurico Dias <[EMAIL PROTECTED]>''Subject: Re: [obm-l] trigonometria''To: obm-l@mat.puc-rio.br''Reply-To: obm-l@mat.puc-rio.br''Essa quest!
ao é
 identica a 12a. da prova da Unicamp de''2003... Como o objetivo é encontrar valores inteiros e''positivos para as tangentes não há motivo para tantos''devaneios... É pura teoria dos números...Eurico Dias'' ''''Start your day with Yahoo! - make it your home page ''http://www.yahoo.com/r/hs '' ''=''Instruções para entrar na lista, sair da lista e usar a lista em''http://www.mat.puc-rio.br/~nicolau/olimp/obm-l.html''==Instruções para entrar na lista, sair da lista e usar a lista
 emhttp://www.mat.puc-rio.br/~nicolau/olimp/obm-l.html=
		Yahoo! Acesso Grátis: Internet rápida e grátis. Instale o discador agora!

Re: [obm-l] trigonometria

2005-08-18 Por tôpico Jefferson Franca


''Devo concordar com o Márcio, realmente não tinha parado pra pensar dessa forma.
Valeu'' ''''Start your day with Yahoo! - make it your home page ''http://www.yahoo.com/r/hs '' ''=''Instruções para entrar na lista, sair da lista e usar a lista em''http://www.mat.puc-rio.br/~nicolau/olimp/obm-l.html''==Instruções para entrar na lista, sair da lista e usar a lista emhttp://www.mat.puc-rio.br/~nicolau/olimp/obm-l.html=
		Yahoo! Acesso Grátis: Internet rápida e grátis. Instale o discador agora!

Re: [obm-l] trigonometria

2005-08-16 Por tôpico Jefferson Franca
Meu caro amigo e ex aluno Eurico, vc não deve ter pensado no seguinte:
a + b + c = a.b.c = (1/bc) + (1/ac) + (1/ab) = 1 e, suponha que bc = 3, ac = 3 e ab = 3, ese tivéssemos bc = 2, ac = 4 e ab = 4?
Eu, sinceramente acho queesta questão não tem só uma solução como vc afirma. 
Valeu e abraço Antonio Eurico Dias [EMAIL PROTECTED] escreveu:

E completando o raciocinio do Dirichlet:
TgA + TgB + TgC = TgA. TgB. TgC
a + b + c = a.b.c = abc - a = b + c = a (bc - 1) = b + c
a = (b + c) / (bc - 1), como a é inteiro positivo: b + c = bc - 1 = bc - b = c + 1
b.(c - 1) = c + 1 = b = (c+1)/(c-1) = b =1 + 2/(c-1)
Logo, como b também é inteiro c - 1 =2 ou c -1 = 1, que levam ao unico conjunto de solucoes inteiras positivas 
(1 , 2 , 3)...

Eurico
Sistema Elite de Ensino - Unidade Belém



Start your day with Yahoo! - make it your home page __Converse com seus amigos em tempo real com o Yahoo! Messenger http://br.download.yahoo.com/messenger/ 

Re: [obm-l] trigonometria

2005-08-16 Por tôpico Jefferson Franca
Obrigado , mas acho que vc não analisou direito esta questão e perceba que ela é sim de trigonometria!!!
Valeu e um abraçoJohann Peter Gustav Lejeune Dirichlet [EMAIL PROTECTED] escreveu:
Ue, pelo menos um dos caras nao e maior que 2 ( o casodo 1 ai escrito, e do 2, que nao e maior ue 2 pois eigual...).E alias vamos fazer logo isso antes que nao de mais!Temos 1/xy+1/xz+1/yz=1. Se xy=c, xz=b, yz=a, temos1/a+1/b+1/c=1Suponha a=b=c. Entao se c=4, temos1/a+1/b+1/c=3/41, absurdo!Logo c=3Ai e so testar!*c=31/a+1/b=2/3Se b=4 entao 1/a+1/b = 2/4 = 1/2  2/3, nao da!Entao b=3Mas 3=c=b=3, o que da b=c=3E (a,b,c)=(3,3,3) da certo.*c=2Ai 1/a+1/b=1/2E entao b=4Testa de novo!**b=41/a=1/4 e ai a=4, (a,b,c)=(4,4,2)**b=31/a=1/6(a,b,c)=(6,3,2)**b=21/a=0. Nao da!c=1, nao serve!Ai e so transformar cada a,b,c em x,y,z:(xy,xz,yz)=(3,3,3) (xyz)^2=27(xy,xz,yz)=(4,4,2)
 (xyz)^2=32(xy,xz,yz)=(6,3,2) (xyz)^2=36=6^2A primeira nao serve (3 nao e quadrado perfeito).A segunda tambem nao...Ja temos entao xyz=6, e agora sem dificuldadecomclui-se que a solucao apresentada anteriormente eunica (alias e exatamente a solucao que voce achou esatisfaz uma porrada de requisitos adicionais...)P.S.: Como eu ja desconfiava este problema nao temnada de trigonometria!--- Jefferson Franca <[EMAIL PROTECTED]>escreveu: E se tivermos, por exemplo, X = 3, Y= 2 e Z = 1 ?  Johann Peter Gustav Lejeune Dirichlet <[EMAIL PROTECTED]>escreveu:tg C= tg A + tg B/ 1- tg A. tgB tg C - tg A . tg B .tg C = tg A+tg B tg A . tg B .tg C = tg A+ tg B + tg C  Ou seja, se acharmos X,Y,Z tais que  XYZ=X+Y+Z, o problema acaba.  Isto e algo facil de resolver, e prova que a solucao e!
 mesmo
 unica. Como o Caio ja disse, e facil ver que pelo menos um dos caras X,Y,Z e no maximo 2.  --- Jefferson Franca  escreveu:   Desculpe, Caio,mas desconfio que não seja.[EMAIL PROTECTED] escreveu:  Suponha que 2 dessas tangentes sejam maiores que 2,  ou seja,  tg A = 2 + x  tg B = 2 + y (x,y 0)A + B + C = 180  A + B = 180 -C   tg (A + B ) = - tg C  tg A + tg B/ 1- tg A. tgB = (4+x+y)/1- (4 + 2(x+y)  +xy)  = (4 + x + y )/ -(3 + 2(x+y) +xy)tg C = (4+x+y)/(3+ 2x + 2y +xy)Teremos que tg C  2 =  4 + x + y  6 + 4x + 4y + 2xy  = 2 + 3x + 3y + 2xy0   Como por hipótese, x e y sao positivos , essa soma  nunca é negativa!
t; 
 ou seja, nunca vale que tg C  2ou seja, é impossível ter as 3 tangentes maior que 2  (simultaneamente)Ou seja, a solução dada pelo nosso amigo é unica!''-- Mensagem Original --  ''Date: Wed, 10 Aug 2005 12:19:32 -0700  ''From: Marcio ''To: obm-l@mat.puc-rio.br  ''Subject: Re: [obm-l] trigonometria  ''Reply-To: obm-l@mat.puc-rio.br  ''  ''  ''On Wed, 10 Aug 2005 06:20:13 -0700, Jefferson  Franca   '' wrote:  ''  '' Será que alguém já viu esta questão ou tem  alguma idéia de como resolver  ''   '' ?  '' Sejam a ,!
b e c
 ângulos internos de umtriângulo  e, supondo que as '' tangentes dos três ângulos sejam números  inteiros e positivos, calcule  ''   '' essas tangentes.  '' Valeu  ''  ''  __  '' Converse com seus amigos em tempo real com o  Yahoo! Messenger  '' http://br.download.yahoo.com/messenger/  ''  ''  ''  ''--   ''Using Opera's revolutionary e-mail client:  http://www.opera.com/mail/  ''  ''Oi, Jefferson.  ''  ''Se não errei nada, aqui vai.  ''  ''Ângulos: a, b e c  ''  ''a + b + c = 180 = tg!
(a + b +
 c)= tg 180, ou  seja, tg(a + b + c) = 0  ''  ''Daí, tg(a + b) + tg(c) = 0.  ''  ''No final das contas, chega-se a  ''  ''tg a + tg b + tg c = (tg a)(tg b)(tg c)  ''  ''Como as tangentes são números inteiros e  positivos, uma opção (não sei  se  ''   ''única) é  ''  ''tg a = 1, tg b = 2 e tg c = 3  ''  ''  ''[]s,  ''  ''Márcio. ''=  ''Instruções para entrar na lista, sair da lista e  usar a lista em ''http://www.mat.puc-rio.br/~nicolau/olimp/obm-l.html
 ''=   =  Instruções para entrar na lista, sair da lista e  usar a lista em  http://www.mat.puc-rio.br/~nicolau/olimp/obm-l.html =-  Yahoo! Acesso Grátis: Internet rápida e grátis.  Instale o discador agora!   __ Converse com seus amigos em tempo real com o Yahoo! Messenger  http://br.download.yahoo.com/messenger/ = Ins!
truções
 para entrar na lista, sair da lista e usar a lista em http://www.mat.puc-rio.br/~nicolau/olimp/obm-l.html=- Yahoo! Acesso Grátis: Internet rápida e grátis. Instale o discador agora!__Converse com se

[obm-l] fração

2005-08-15 Por tôpico Jefferson Franca
Como posso provar : n é um número natural e primo.Prove que se a representação decimal de n tiver n - 1 casas decimais, então seu período é (10^(n - 1))/ n ?
Valeu__Converse com seus amigos em tempo real com o Yahoo! Messenger http://br.download.yahoo.com/messenger/ 

Re: [obm-l] trigonometria

2005-08-15 Por tôpico Jefferson Franca
Desculpe, Caio,mas desconfio que não seja.[EMAIL PROTECTED] escreveu:
Suponha que 2 dessas tangentes sejam maiores que 2, ou seja,tg A = 2 + xtg B = 2 + y (x,y 0)A + B + C = 180A + B = 180 -C tg (A + B ) = - tg Ctg A + tg B/ 1- tg A. tgB = (4+x+y)/1- (4 + 2(x+y) +xy)= (4 + x + y )/ -(3 + 2(x+y) +xy)tg C = (4+x+y)/(3+ 2x + 2y +xy)Teremos que tg C  2 =4 + x + y  6 + 4x + 4y + 2xy= 2 + 3x + 3y + 2xy0 Como por hipótese, x e y sao positivos , essa soma nunca é negativaou seja, nunca vale que tg C  2ou seja, é impossível ter as 3 tangentes maior que 2 (simultaneamente)Ou seja, a solução dada pelo nosso amigo é unica!''-- Mensagem Original --''Date: Wed, 10 Aug 2005 12:19:32 -0700''From: Marcio <[EMAIL PROTECTED]>''To: obm-l@mat.puc-rio.br''Subject: Re: [obm-l]
 trigonometria''Reply-To: obm-l@mat.puc-rio.br''On Wed, 10 Aug 2005 06:20:13 -0700, Jefferson Franca ''<[EMAIL PROTECTED]> wrote: Será que alguém já viu esta questão ou tem alguma idéia de como resolver'' '' ?'' Sejam a ,b e c ângulos internos de umtriângulo e, supondo que as '' tangentes dos três ângulos sejam números inteiros e positivos, calcule'' '' essas tangentes.'' Valeu __'' Converse com seus amigos em tempo real com o Yahoo! Messenger'' http://br.download.yahoo.com/messenger/-- ''Using Opera's revolutionary e-mail client: http://www.opera.com/mail/Oi, Jefferson.Se não errei nada, aqui vai.Ângulos: a, b e
 ca + b + c = 180 = tg(a + b + c)= tg 180, ou seja, tg(a + b + c) = 0Daí, tg(a + b) + tg(c) = 0.No final das contas, chega-se atg a + tg b + tg c = (tg a)(tg b)(tg c)Como as tangentes são números inteiros e positivos, uma opção (não seise'' ''única) étg a = 1, tg b = 2 e tg c = 3''[]s,Márcio.''=''Instruções para entrar na lista, sair da lista e usar a lista em''http://www.mat.puc-rio.br/~nicolau/olimp/obm-l.html''==Instruções para entrar na lista, sair da lista e usar a lista
 emhttp://www.mat.puc-rio.br/~nicolau/olimp/obm-l.html=
		Yahoo! Acesso Grátis: Internet rápida e grátis. Instale o discador agora!

Re: [obm-l] fração

2005-08-15 Por tôpico Jefferson Franca
Desculpa, errei feio, Quis dizer 1/n tem n - 1 casas decimais.Johann Peter Gustav Lejeune Dirichlet [EMAIL PROTECTED] escreveu:
Como assim, a representacao de n tem n-1 casasdecimais?Isso quer dizer que esta representacao e finita??--- Jefferson Franca <[EMAIL PROTECTED]>escreveu: Como posso provar : n é um número natural e primo.Prove que se a representação decimal de n tiver n - 1 casas decimais, então seu período é (10^(n - 1))/ n ? Valeu  __ Converse com seus amigos em tempo real com o Yahoo! Messenger  http://br.download.yahoo.com/messenger/ __Converse com seus amigos em tempo real com o Yahoo! Messenger http://br.download.yahoo.com/messenger/ =Instruções para entrar na lista, sair da lista e usar!
 a lista
 emhttp://www.mat.puc-rio.br/~nicolau/olimp/obm-l.html=__Converse com seus amigos em tempo real com o Yahoo! Messenger http://br.download.yahoo.com/messenger/ 

Re: [obm-l] trigonometria

2005-08-15 Por tôpico Jefferson Franca
E se tivermos, por exemplo, X = 3, Y= 2 e Z = 1 ?Johann Peter Gustav Lejeune Dirichlet [EMAIL PROTECTED] escreveu:
tg C= tg A + tg B/ 1- tg A. tgBtg C - tg A . tg B .tg C = tg A+tg Btg A . tg B .tg C = tg A+ tg B + tg COu seja, se acharmos X,Y,Z tais que XYZ=X+Y+Z, o problema acaba.Isto e algo facil de resolver, e prova que a solucao emesmo unica.Como o Caio ja disse, e facil ver que pelo menos umdos caras X,Y,Z e no maximo 2.--- Jefferson Franca <[EMAIL PROTECTED]>escreveu: Desculpe, Caio,mas desconfio que não seja.  [EMAIL PROTECTED] escreveu: Suponha que 2 dessas tangentes sejam maiores que 2, ou seja, tg A = 2 + x tg B = 2 + y (x,y 0)  A + B + C = 180 A + B = 180 -C  tg (A + B ) = - tg C tg A + tg B/ 1- tg A. tgB = (4+x+y)/1- (4 + 2(x+y) +xy) = (4 + x + y )/ -(3 + 2(x+y) +xy)  tg C = (4+x+y)/(3+ !
2x + 2y
 +xy)  Teremos que tg C  2 = 4 + x + y  6 + 4x + 4y + 2xy = 2 + 3x + 3y + 2xy0  Como por hipótese, x e y sao positivos , essa soma nunca é negativa ou seja, nunca vale que tg C  2  ou seja, é impossível ter as 3 tangentes maior que 2 (simultaneamente)Ou seja, a solução dada pelo nosso amigo é unica!  ''-- Mensagem Original -- ''Date: Wed, 10 Aug 2005 12:19:32 -0700 ''From: Marcio   ''To: obm-l@mat.puc-rio.br ''Subject: Re: [obm-l] trigonometria ''Reply-To: obm-l@mat.puc-rio.br '' '' ''On Wed, 10 Aug 2005 06:20:13 -0700, Jefferson Franca  '' wrote: '' '' Será que alguém já viu esta questão ou tem alguma idéia de como
 resolver ''  '' ? '' Sejam a ,b e c ângulos internos de umtriângulo e, supondo que as   '' tangentes dos três ângulos sejam números inteiros e positivos, calcule ''  '' essas tangentes. '' Valeu '' '' __ '' Converse com seus amigos em tempo real com o Yahoo! Messenger '' http://br.download.yahoo.com/messenger/ '' '' '' ''--  ''Using Opera's revolutionary e-mail client: http://www.opera.com/mail/ '' ''Oi, Jefferson. '' ''Se não errei nada, aqui vai. '' ''Ângulos: a, b e c '' ''a + b + c = 180 = tg(a + b + c)= tg 180, ou seja, tg(a + b + c) = 0
 '' ''Daí, tg(a + b) + tg(c) = 0. '' ''No final das contas, chega-se a '' ''tg a + tg b + tg c = (tg a)(tg b)(tg c) '' ''Como as tangentes são números inteiros e positivos, uma opção (não sei se ''  ''única) é '' ''tg a = 1, tg b = 2 e tg c = 3 '' '' ''[]s, '' ''Márcio.''= ''Instruções para entrar na lista, sair da lista e usar a lista em''http://www.mat.puc-rio.br/~nicolau/olimp/obm-l.html''=   = Instruções para entrar na lista, sair da lista e!

 usar a lista em http://www.mat.puc-rio.br/~nicolau/olimp/obm-l.html=- Yahoo! Acesso Grátis: Internet rápida e grátis. Instale o discador agora!__Converse com seus amigos em tempo real com o Yahoo! Messenger http://br.download.yahoo.com/messenger/ =Instruções para entrar na lista, sair da lista e usar a lista emhttp://www.mat.puc-rio.br/~nicolau/olimp/obm-l.html=
		Yahoo! Acesso Grátis: Internet rápida e grátis. Instale o discador agora!

[obm-l] trigonometria

2005-08-10 Por tôpico Jefferson Franca
Será que alguém já viu esta questão ou tem alguma idéia de como resolver ?
Sejam a ,b e c ângulos internos de umtriângulo e, supondo que as tangentes dos três ângulos sejam números inteiros e positivos, calcule essas tangentes.
Valeu__Converse com seus amigos em tempo real com o Yahoo! Messenger http://br.download.yahoo.com/messenger/ 

Re: [obm-l] [off-topic?] provas do IME - versao 6

2005-07-19 Por tôpico Jefferson Franca
Bom dia amigo Sérgio e demais amigos da lista. Minha dúvida é quanto ao site em que estão disponibilizadas as provas do ime. Tentei mas não conseguir entrar.
O site é mesmo www.lps.ufr.br/~sergioln/ime ?Sergio Lima Netto [EMAIL PROTECTED] escreveu:
Caros colegas da lista,Faltou luz na UFRJ este fim-de-semana. Como os servidoresdo LPS nao estao configurados para retornar automaticamete,alguns de voces nao conseguiram baixar a versao 6 das provas do IME.Peco desculpas. O problema foi solucionado esta manhae pelo que acabei de testar nao deve haver mais problema.Quem tiver interesse no arquivo, e' so' acessar o sitewww.lps.ufr.br/~sergioln/imeComo disse antes, a versao atual tem 1.1 MB e acho que precisado Acrobat6 [mas abriu sem problemas (aparentemente) no Acrobat5la' de casa].Retorno, agradecidamente tambem, os "agradecimentos"e os elogios recebidos. Abraco,sergio=Instruções para entrar na lista, sair da lista e usar a lista
 emhttp://www.mat.puc-rio.br/~nicolau/olimp/obm-l.html=
		Yahoo! Acesso Grátis: Internet rápida e grátis. Instale o discador agora!

Re: [obm-l] [off-topic?] provas do IME - versao 6

2005-07-19 Por tôpico Jefferson Franca
Pensei que estivesse faltando o '"j" e depois que tentei de novo(colocando o j) conseguir entrar.
Muito obrigado , seu trabalho é muito importante para tirar dúvidas e também para o crescimento intelectual .
Valeu SérgioSergio Lima Netto [EMAIL PROTECTED] escreveu:
Caro Jefferson e demais colegas da lista,o site correto e'www.lps.ufrj.br/~sergioln/imeNa mensagem anterior, (que o Jefferson Franca usou)faltou o "j" em "ufrj".Mais uma vez tenho que pedir desculpas.As vezes o site sai do ar, em geral um fim-de-semana ou outro.Durante a semana o site e' bem "estavel".Nao sei se cabe discutir aqui nesta listaas solucoes que eu coloco. Ate' pq, um pouco pelagrandiosidade do material e muito pela minha incapacidadede enxergar os meus proprios erros, o material deveconter alguns (varios?) erros mesmo. Ainda mais,eu costumo algebrizar os problemas geometricos e namaioria das vezes a minha solucao nao e' a maissimples/elegante/eficiente etc.Apontar os erros da versao6 nesta lista,eu acho que vai encher a paciencia de muitas pessoas boasque nao estao int!
eressadas
 nisto.Porem, eu preciso que voces me indiquem os erros para corrigi-losnuma futura versao. Se eu puder, eu gostaria de solicitar que aspessoas entrem em contato direto comigo[EMAIL PROTECTED]ao inves de atraves da lista. Sempre que posso, eu procuro responder.As vezes, demora um pouco, mas eu respondo sim.O unico problema e' que eu recebo tanto spam que se a mensagemnao estiver com um "subject" bem definido (a palavra IME ajuda),eu acabo inadvertidamente apagando a mensagem.Agradeco antecipadamente e sinceramente a colaboracao(duvidas, criticas, comentarios, novas provas, erros encontrados)de todos.Abraco,sergioOn Tue, 19 Jul 2005, Jefferson Franca wrote: Bom dia amigo Sérgio e demais amigos da lista. Minha dúvida é quanto ao site em que estão disponibilizadas as provas do ime. Tentei mas não conseguir entrar. O site é mesmo www.lps.ufr.br/~sergioln/ime ? &!
gt;
 Sergio Lima Netto <[EMAIL PROTECTED]>escreveu: Caros colegas da lista, Faltou luz na UFRJ este fim-de-semana. Como os servidores do LPS nao estao configurados para retornar automaticamete, alguns de voces nao conseguiram baixar a versao 6 das provas do IME. Peco desculpas. O problema foi solucionado esta manha e pelo que acabei de testar nao deve haver mais problema. Quem tiver interesse no arquivo, e' so' acessar o site  www.lps.ufr.br/~sergioln/ime  Como disse antes, a versao atual tem 1.1 MB e acho que precisa do Acrobat6 [mas abriu sem problemas (aparentemente) no Acrobat5 la' de casa].  Retorno, agradecidamente tambem, os "agradecimentos" e os elogios recebidos. Abraco, sergio  = Instruções para entrar na lista, sair da lista e usar a lis!
ta
 em http://www.mat.puc-rio.br/~nicolau/olimp/obm-l.html =   - Yahoo! Acesso Grátis: Internet rápida e grátis. Instale o discador agora!=Instruções para entrar na lista, sair da lista e usar a lista emhttp://www.mat.puc-rio.br/~nicolau/olimp/obm-l.html=
		Yahoo! Acesso Grátis: Internet rápida e grátis. Instale o discador agora!

Re: [obm-l] FUNÇÕES SOBREJETORAS

2005-06-06 Por tôpico Jefferson Franca
1)Para f ser sobrejetora é necessário que o conjunto imagem seja exatamente igual ao contradomínio, ou seja, B é um intervalo fechado à esquerda em 3 e aberto à direita para mais infinito2) Agora B tem que ser um intervalo fechado àesquerda em y = - 3 e aberto à direita para mais infinitoMiguel Mossoro [EMAIL PROTECTED] escreveu:

Olá pessoal, 

Gostaria de uma ajuda nesses 2 exercícios:
1) Seja f:R-B definida por f(x) = x^2 - 2x + 4. Determine B para que f seja sobrejetora.
e
2) Seja a função de A = { xE R | -5 = x  2 } em B C R, definida por f(x) = |x + 3| - 2. Se f é sobrejetora, determine B.
Grato


Yahoo! Mail: agora com 1GB de espaço grátis. Abra sua conta!
		Yahoo! Mail: agora com 1GB de espaço grátis. Abra sua conta!

Re: [obm-l] Ainda sobre livros...

2005-05-17 Por tôpico Jefferson Franca
Vamos continuar esse papo em PVT.

Araray Velho [EMAIL PROTECTED] escreveu:
Exatamente, Jefferson,Eu também tinha visto aquela questão da elipse no Caronnet, bem comoaquela de logaritmo ( que estavam em PA ) no Lidsky e, há poucos dias,encontrei a 9 ( do determinante ) no livro do Proskuriakov. Ou seja,por baixo já são 3 questões literalmente copiadas de livros não tãopouco conhecidos. Se duvidar, mais questões da prova seriam de outroslivros. Além disso, eu considero essas 3 questões como sendo as maisinteressantes da prova.Há poucos dias, também, um amigo meu que passara no IME e chegou a irpara lá no início do ano ( retornando pouco tempo depois ) me deu umalista de exercícios de limites, derivadas e integrais. Fiquei pasmo aover que a lista era uma cópia ( mais de 90% dos problemas ) do livrodo Demidovitch e o autor da lista de exercícios não fez qualquercitação ao livro fonte. Ao meu ve!
r, isso é
 terrível. Enfim, não estouaqui para julgar uma ou outra instituição e, muito menos, uma ou outrapessoa. Estou apenas fazendo constatações. Há a possibilidade (confesso que a vejo como extremamente remota ) de que tenha havido umacoincidência.Podemos continuar esse assunto em PVT ? Apenas para não fugirmos dosprincípios da lista !Sinceramente,-- Araray Velho[EMAIL PROTECTED]ICQ 20464041MSN [EMAIL PROTECTED]On 5/14/05, Jefferson Franca <[EMAIL PROTECTED]>wrote: Araray, Eu tenho a impressâo de que conheci vc em Porto Alegre ano passado no elite! Procure na amazon ou em alguns sites especializados nos livros da mir que a meu ver sâo espetaculares. Sabe aquela questâo de elipse da prova do ano passado do ime? Está no caronnet e eu , no dia anterior resolvi a mesma questâo para alguns alunos, sorte, muita sorte! Acho que uma instituiçâo séria e com!
 nome a
 zelar como o IME deveria elaborar melhor seu vestibular, as provas deveriam ser inéditas. Um abraço  Araray Velho <[EMAIL PROTECTED]>escreveu: Jefferson,   Agradeço a tua resposta.  Sim, conheço o Bagual, fiz um ano de Elite também. Meu interesse não é ter uma base para alguma finalidade em especial, mas apenas por interesse. Quanto aos livros, são muito bons mesmo, mas estão restritos a pouco conteúdo. Além disso, sempre estou em busca de novas bibliografias. Achei alguns títulos novos muitíssimo interessantes. Como o "Problemas de álgebra linear" do Proskuriakov. Há problemas semelhantes ao problema 9 de matemática do último vestibular do IME ( ver problema 374 ). Há mais tantos outros livros com bons problemas. Posso passar o restante das novas bibliografias que achei. De qualquer forma, tenho material para me divertir até!
 o final
 do ano. Livros de problema são um prazer que eu me dou, mas não os vejo como um estudo. Como estudo, eu já tenho muito que me preocupar c! om o curso de física ( onde eu já deveria estar formado e, ainda, faltam alguns semestres ).  Sinceramente,  --  Araray Velho [EMAIL PROTECTED] ICQ 20464041 MSN [EMAIL PROTECTED]   On 5/12/05, Jefferson Franca wrote:  Meu camarada esses livros são espetaculares!  Se vc estudar direito por esses livros terá uma ótima base. Vc conhece o  Bagual? Pede ajuda e orientação pra ele.  Um abraçoAraray Velho escreveu:  Pessoal,  Peço desculpas por mais uma mensagem sobre livros. Eu gostaria de  receber sugestões de livros de exercícios e problemas em matemática ou  física em qualquer!
 área e
 nível.Eu tenho e conheço apenas o Lidsky ( Problemas de matemática Elementar  ), Saraeva ( Problemas Selecionados de Física Elementar ), Demidovitch  ( Problemas e exercícios de análise matemática ) e o ! Caronnet (  Problemas de Geometria ).Há outras coleções de problemas e exercícios como essas ?Agradeço, desde já, a todos pela atenção e compreensão.Abraços,--   Araray Velho  [EMAIL PROTECTED]  ICQ 20464041  MSN [EMAIL PROTECTED]=  Instruções para entrar na lista, sair da lista e usar a lista em  http://www.mat.puc-rio.br/~nicolau/olimp/obm-l.html 
 =  __  Converse com seus amigos em tempo real com o Yahoo! Messenger   http://br.download.yahoo.com/messenger/--  Araray Velho [EMAIL PROTECTED] ICQ 20464041 MSN [EMAIL PROTECTED]  = Instruções para entrar na lista, sair da lista e usar a lista em http://www.mat.puc-rio.br/~nicolau/olimp/obm-l.html =    Yahoo! Mail: agora com 1GB de espaço grátis. Abra sua conta!   -- Araray Velho[EMAIL PROTECTED]ICQ 20464041MSN
 [EMAIL PROTECTED]=Instruções para entrar na lista, sair da lista e usar a lista emhttp://www.mat.puc-rio.br/~ni

Re: [obm-l] Ainda sobre livros...

2005-05-14 Por tôpico Jefferson Franca
Araray,
Eu tenho a impressâo de que conheci vc em Porto Alegre ano passado no elite!
Procure na amazon ou em alguns sites especializados nos livros da mir que a meu ver sâo espetaculares.
Sabe aquela questâo de elipse da prova do ano passado do ime?
Está no caronnet e eu , no dia anterior resolvi a mesma questâo para alguns alunos, sorte, muita sorte!
Acho que uma instituiçâo séria e com nome a zelar como o IME deveria elaborar melhor seu vestibular, as provas deveriam ser inéditas.
Um abraçoAraray Velho [EMAIL PROTECTED] escreveu:
Jefferson,Agradeço a tua resposta. Sim, conheço o Bagual, fiz um ano de Elite também.Meu interesse não é ter uma base para alguma finalidade em especial,mas apenas por interesse. Quanto aos livros, são muito bons mesmo, masestão restritos a pouco conteúdo. Além disso, sempre estou em busca denovas bibliografias.Achei alguns títulos novos muitíssimo interessantes. Como o "Problemasde álgebra linear" do Proskuriakov. Há problemas semelhantes aoproblema 9 de matemática do último vestibular do IME ( ver problema374 ). Há mais tantos outros livros com bons problemas. Posso passar orestante das novas bibliografias que achei.De qualquer forma, tenho material para me divertir até o final do ano.Livros de problema são um prazer que eu me dou, mas não os vejo comoum estudo. Como estudo, eu já tenho muito que me preocupar c!
om o
 cursode física ( onde eu já deveria estar formado e, ainda, faltam algunssemestres ).Sinceramente,-- Araray Velho[EMAIL PROTECTED]ICQ 20464041MSN [EMAIL PROTECTED]On 5/12/05, Jefferson Franca <[EMAIL PROTECTED]>wrote: Meu camarada esses livros são espetaculares! Se vc estudar direito por esses livros terá uma ótima base. Vc conhece o Bagual? Pede ajuda e orientação pra ele. Um abraço  Araray Velho <[EMAIL PROTECTED]>escreveu: Pessoal,   Peço desculpas por mais uma mensagem sobre livros. Eu gostaria de receber sugestões de livros de exercícios e problemas em matemática ou física em qualquer área e nível.  Eu tenho e conheço apenas o Lidsky ( Problemas de matemática Elementar ), Saraeva ( Problemas Selecionados de Física Elementar ), Demidovitch ( Problemas e exercícios de análise matemática ) e o !
Caronnet
 ( Problemas de Geometria ).  Há outras coleções de problemas e exercícios como essas ?  Agradeço, desde já, a todos pela atenção e compreensão.  Abraços,  --  Araray Velho [EMAIL PROTECTED] ICQ 20464041 MSN [EMAIL PROTECTED]  = Instruções para entrar na lista, sair da lista e usar a lista em http://www.mat.puc-rio.br/~nicolau/olimp/obm-l.html =   __ Converse com seus amigos em tempo real com o Yahoo! Messenger  http://br.download.yahoo.com/messenger/ -- Araray Velho[EMAIL PROTECTED]ICQ 20464041MSN
 [EMAIL PROTECTED]=Instruções para entrar na lista, sair da lista e usar a lista emhttp://www.mat.puc-rio.br/~nicolau/olimp/obm-l.html=
		Yahoo! Mail: agora com 1GB de espaço grátis. Abra sua conta!

Re: [obm-l] Ainda sobre livros...

2005-05-12 Por tôpico Jefferson Franca
Meu camarada esses livros são espetaculares!
Se vc estudar direito por esses livros terá uma ótima base. Vc conhece o Bagual? Pede ajuda e orientação pra ele.
Um abraçoAraray Velho [EMAIL PROTECTED] escreveu:
Pessoal,Peço desculpas por mais uma mensagem sobre livros. Eu gostaria dereceber sugestões de livros de exercícios e problemas em matemática oufísica em qualquer área e nível.Eu tenho e conheço apenas o Lidsky ( Problemas de matemática Elementar), Saraeva ( Problemas Selecionados de Física Elementar ), Demidovitch( Problemas e exercícios de análise matemática ) e o Caronnet (Problemas de Geometria ).Há outras coleções de problemas e exercícios como essas ?Agradeço, desde já, a todos pela atenção e compreensão.Abraços,-- Araray Velho[EMAIL PROTECTED]ICQ 20464041MSN [EMAIL PROTECTED]=Instruções para entrar na lista, sair da lista e usar a lista
 emhttp://www.mat.puc-rio.br/~nicolau/olimp/obm-l.html=__Converse com seus amigos em tempo real com o Yahoo! Messenger http://br.download.yahoo.com/messenger/ 

Re: [obm-l] Livros

2005-01-19 Por tôpico Jefferson Franca
Oi! Boa sorte na sua empreitada.
Acho que vc dveria utilizar as apostilas e listas de exercícios do sistema elite de ensino. Se vc estiver interessado escreva para [EMAIL PROTECTED]
[EMAIL PROTECTED] ou [EMAIL PROTECTED]
O primeiro é da unidade em Porto Alegre, os dois últimos são de diretores .
Não custa nada, vc trocar informaçoes sobre material did´[atico deles(eu achei muito bom!) e além disso, eles são iteanos!
Um abraçoFelipe Nardes [EMAIL PROTECTED] wrote:
vou fazer a turma ITA do poliedro esse ano e gostaria que vocês me indicassem bom livros de matemática, física e química para complementar as apostilas que o curso utiliza. Já tenho os livros do 2º grau, por isso queria pegar alguns livros melhores, com exercícios mais difíceis e uma teoria mais completa.valeu_MSN Messenger: converse online com seus amigos . http://messenger.msn.com.br=Instruções para entrar na lista, sair da lista e usar a lista emhttp://www.mat.puc-rio.br/~nicolau/olimp/obm-l.html=
		Yahoo! Acesso Grátis - Internet rápida e grátis. Instale o discador do Yahoo! agora.

Re: [obm-l] oi!

2005-01-09 Por tôpico Jefferson Franca
Oi kellem, seja bem vinda. Sou o Jefferson do elite belém que agora tá no elite tijuca. 
Com certeza vc vai encontrar muita gente conhecida aqui!
Um abraço"Kellem :-) 100% SeJ" [EMAIL PROTECTED] wrote:
Oi GenteBem, meu nome é Kellem e sou do RJ. Fazia UFF e, agora q passeo pro IME epro ITA, vou pra um dos dois, só q, até agora, fico sem dormir sem saber o qfazer, pq tb faço Iniciação Científica no IMPA! Eu não sei se fico praarriscar o IMPA... O problema é q gosto muito de matemática, mas acho q nãosei mto não (sempre acho q não sei nada e q vou tirar zero). Enfim, tôaqui, e espero poder ajudar e obter ajuda nos probleminhas, tá?Alguém daqui me conhece (ou eu conheço??)?BJãoKellem=Instruções para entrar na lista, sair da lista e usar a lista emhttp://www.mat.puc-rio.br/~nicolau/olimp/obm-l.html=__Co!
nverse
 com seus amigos em tempo real com o Yahoo! Messenger http://br.download.yahoo.com/messenger/ 

[obm-l] =?iso-8859-1?q?D=DAVIDA=20em=20an=E1lise?=

2004-12-20 Por tôpico Jefferson Franca
Tenho 2 dúvidas: 1) estava estudando análise no livro 
do Djairo Guedes  e ele afirma que o conjunto dos
racionais é um corpo, enquanto que o dos inteiros não
é um corpo, bem, se eu entendi direito, um conjunto
pra ser  cosiderado um corpo
tem que satisfazer o seguinte: a adição e a
multiplicação têm que estar definidas para todos seus
elementos, isto é, se x e y peretencem a um conjunto
E, então x + y tbm pertence ao conjunto E e para os
elementos x e y ,peretencentes a E, o número xy
pertence ao conjunto E, diante disso,entendo que o
conjunto dos inteiros é um corpo!
2) como demonstrar que oconjunto dos é denso em R, ou
seja, como provar que ,dados 2 reais, x e y, com x 
y, existem raciomais q tais que x  q  y ?





___ 
Yahoo! Acesso Grátis - Instale o discador do Yahoo! agora. http://br.acesso.yahoo.com/ 
- Internet rápida e grátis
=
Instruções para entrar na lista, sair da lista e usar a lista em
http://www.mat.puc-rio.br/~nicolau/olimp/obm-l.html
=


[obm-l] =?iso-8859-1?q?=20An=E1lise?=

2004-12-20 Por tôpico Jefferson Franca


Tenho 2 dúvidas: 1) estava estudando análise no livro 
do Djairo Guedes  e ele afirma que o conjunto dos
racionais é um corpo, enquanto que o dos inteiros não
é um corpo, bem, se eu entendi direito, um conjunto
pra ser  cosiderado um corpo
tem que satisfazer o seguinte: a adição e a
multiplicação têm que estar definidas para todos seus
elementos, isto é, se x e y peretencem a um conjunto
E, então x + y tbm pertence ao conjunto E e para os
elementos x e y ,peretencentes a E, o número xy
pertence ao conjunto E, diante disso,entendo que o
conjunto dos inteiros é um corpo!
2) como demonstrar que oconjunto dos racionais é denso
em R, ou
seja, como provar que ,dados 2 reais, x e y, com x 
y, existem raciomais q tais que x  q  y ?

__
Converse com seus amigos em tempo real com o Yahoo! Messenger 
http://br.download.yahoo.com/messenger/ 
=
Instruções para entrar na lista, sair da lista e usar a lista em
http://www.mat.puc-rio.br/~nicolau/olimp/obm-l.html
=


=?iso-8859-1?q?Re:=20[obm-l]=20Re:=20[obm-l]=20D=DAVIDA=20em=20an=E1lise?=

2004-12-20 Por tôpico Jefferson Franca
 --- Fernando Villar [EMAIL PROTECTED] escreveu:

 Para mostrar que entre dois números reais existe um
 racional vamos mostrar o
 caso geral
 
 Sejam x e y dois números reais tais que xy.
 Considere d= x-y 0.
 
 Como a sequencia (1/n) converge a zero, existe N tal
 que 01/Nd.
 
 Considere as sequências (n/m) e (-n/m) indexadas por
 n.
 Ambas divergem, a primeira para +inf e a segunda
 para -inf. em qualquer dois
 casos podemos garantir que algum dos seus termos
 pertence a intervalo (y,x).
 Para isso analise o caso de x0 ou y0. O caso y0 e
 x0 é direto pois 0
 está entre eles!
 
 []s
 
 Fernando
 
 
 
 - Original Message - 
 From: Jefferson Franca [EMAIL PROTECTED]
 To: [EMAIL PROTECTED]
 Sent: Monday, December 20, 2004 2:09 PM
 Subject: [obm-l] DÚVIDA em análise
 
 
  Tenho 2 dúvidas: 1) estava estudando análise no
 livro
  do Djairo Guedes  e ele afirma que o conjunto dos
  racionais é um corpo, enquanto que o dos inteiros
 não
  é um corpo, bem, se eu entendi direito, um
 conjunto
  pra ser  cosiderado um
 corpo
  tem que satisfazer o seguinte: a adição e a
  multiplicação têm que estar definidas para todos
 seus
  elementos, isto é, se x e y peretencem a um
 conjunto
  E, então x + y tbm pertence ao conjunto E e para
 os
  elementos x e y ,peretencentes a E, o número xy
  pertence ao conjunto E, diante disso,entendo que o
  conjunto dos inteiros é um corpo!
  2) como demonstrar que oconjunto dos é denso em R,
 ou
  seja, como provar que ,dados 2 reais, x e y, com x
 
  y, existem raciomais q tais que x  q  y ?
 
 
 
 
 
 

___
  Yahoo! Acesso Grátis - Instale o discador do
 Yahoo! agora.
 http://br.acesso.yahoo.com/ - Internet rápida e
 grátis
 

=
  Instruções para entrar na lista, sair da lista e
 usar a lista em
 
 http://www.mat.puc-rio.br/~nicolau/olimp/obm-l.html
 

=
 
  Esta mensagem foi verificada pelo E-mail Protegido
 Terra.
  Scan engine: McAfee VirusScan / Atualizado em
 15/12/2004 / Versão:
 4.4.00 - Dat 4415
  Proteja o seu e-mail Terra:
 http://www.emailprotegido.terra.com.br/
 
  E-mail classificado pelo Identificador de Spam
 Inteligente Terra.
  Para alterar a categoria classificada, visite
 

http://www.terra.com.br/centralunificada/emailprotegido/imail/imail.cgi?+_u=f_villar_l=1,1103559798.947442.27209.mongu.terra.com.br,3068,Des15,Des15
 
 
 
 

=
 Instruções para entrar na lista, sair da lista e
 usar a lista em
 http://www.mat.puc-rio.br/~nicolau/olimp/obm-l.html

=
  Valeu pela ajuda!





___ 
Yahoo! Acesso Grátis - Instale o discador do Yahoo! agora. http://br.acesso.yahoo.com/ 
- Internet rápida e grátis
=
Instruções para entrar na lista, sair da lista e usar a lista em
http://www.mat.puc-rio.br/~nicolau/olimp/obm-l.html
=


Re: [obm-l] IME

2004-10-26 Por tôpico Jefferson Franca
O ime como sempre, copiou questões de livros! por exemplo, esta questão de logaritmo é do lidsky e a questão 7 é do carronet!a questão 3 tem um erro na digitaçãoAriel de Silvio [EMAIL PROTECTED] wrote:
Olá a todos,Começaram hoje as provas do IME. Hoje foi realizada a prova de matemática.Lembro que ano passado propuseram na lista resoluções das questõesdiferentes da resoluções dadas pelos cursinhos. Esse ano vão fazer também?O Poliedro (www.sistemapoliedro.com.br) está resolvendo. O GPI diz que iráresolver também (www.gpi.g12.br). O Poliedro está colocando o enunciado emapenas algumas das questões.Mas já começo com um pedido, a questão 3. Vou passar direto aqui.Sejam a, b, c, d números reais positivos e diferentes de 1. Sabendo quelog[a](d), log[b](d) e log[c](d) são termos consecutivos de um progressãoaritmética, demonstre que:c^2 = (ac)^log[a](d)log[a](d) é log de d na base aSó que ninguém que conversei conseguiu chegar nisso. Apenas em:c^2 = (ac)^log[a](b)Cheguei nisso, e não vejo !
motivo
 para b = dDe resto tiveram questões MUITO simples, outras malvadas e outras realmentedifíceis.A questão 4 por exemplo dava duas equações de quarto grau, pedia as raizescomuns. Porém não tinha raízes comuns! Cruel pra quem tá ali fazendo a prova.[]sAriel=Instruções para entrar na lista, sair da lista e usar a lista emhttp://www.mat.puc-rio.br/~nicolau/olimp/obm-l.html=
		Yahoo! Acesso Grátis - Internet rápida e grátis. Instale o discador agora!

Re: [obm-l] RES:_[obm-l]_Combinatória

2004-07-22 Por tôpico Jefferson Franca
Muito obrigado, Guilherme!Guilherme [EMAIL PROTECTED] wrote:


Olá, Jefferson!

chamando de :

a = número de atletas
t = número de times
k = número de atletas em cada time
n = número de times em que cada atleta participa (resposta da a)
m = número de times em que cada par de atletas fica junto (resposta da b)

Temos:
n = t . k / a pois t . k é o número de atletas inscritos (contando inclusive as repetições). Ao dividir pelo número de atletas, temos o número de vezes que cada atleta aparece.
E ainda, 

C(a, 2) sendo o número de pares de atletas (combinação do número de atletas, dois a dois)
C(k, 2) sendo o número de pares de atletas que podemos formar em cada time, 

Fica: 

C(a,2). m = t . C(k, 2) pois o número de pares de atletas vezes o número de vezes que cada par aparece é igual ao número de times vezes o número de pares por time (o número total de pares, incluindo os repetidos, em cada lado da igualdade).

Logo, m = t . C(k,2) / C(a,2) ou, mais simplesmente, m = [t.k.(k-1)] / [a(a-1)]

As respostas, são, portanto, 

a) t.k/a
b) [t.k.(k-1)] / [a(a-1)]

Espero que esteja certo.

Um grande abraço, 

Guilherme Marques.




-Mensagem original-De: [EMAIL PROTECTED] [mailto:[EMAIL PROTECTED] Em nome de Jeffferson FrancaEnviada em: quinta-feira, 22 de julho de 2004 01:58Para: [EMAIL PROTECTED]Assunto: [obm-l] Combinatória
Ainda não conseguir resolver esta questão e por isso estou sem sossego, será que alguém poderia me ajudar?
A questão é a seguinte: A partir de um conjunto de a atletas formam-se t times de k atletas cada. Todos os atletas participam de um mesmo número de times e cada par de atletas fica junto no mesmo time um mesmo número de vezes. Determine: 
a) De quantos times cada atleta participa 
b) Em quantos times cada par de atletas fica junto


Yahoo! Mail agora ainda melhor: 100MB, anti-spam e antivírus grátis!
		Yahoo! Mail agora ainda melhor: 100MB, anti-spam e antivírus grátis!

[obm-l] Combinatória

2004-07-21 Por tôpico Jefferson Franca
Ainda não conseguir resolver esta questão e por isso estou sem sossego, será que alguém poderia me ajudar?
A questão é a seguinte: A partir de um conjunto de a atletas formam-se t times de k atletas cada. Todos os atletas participam de um mesmo número de times e cada par de atletas fica junto no mesmo time um mesmo número de vezes. Determine: 
a) De quantos times cada atleta participa 
b) Em quantos times cada par de atletas fica junto
		Yahoo! Mail agora ainda melhor: 100MB, anti-spam e antivírus grátis!

[obm-l] Geometria plana e combinatória

2004-07-19 Por tôpico Jefferson Franca
Duas questões que estão tirando o meu sossego são:
01. ABCDE é um pentágono convexo . Mostre que os baricentros dos triângulos ABE, BCE, CDE e DAE formam um quadrilátero convexo cuja área é 2/9 da área do quadrilátero ABCD.
02. A partir de um conjunto de a atletas forman-se t times de k atletas cada. Todos os atletas participam de um mesmo número de times e cada par de atletas fica junto no mesmo time um mesmo número de vezes. Determine:
a) De quantos times cada atleta participa?
b) Em quantos times cada par de atletas fica junto?
Espero a colaboração de alguém na lista para a resolução destas questões e meu consequente sossego.
		Yahoo! Mail agora ainda melhor: 100MB, anti-spam e antivírus grátis!

Re:_[obm-l]_Geometria_plana_e_combinatória

2004-07-19 Por tôpico Jefferson Franca
Valeu, Carlos, mas vc poderia me ensinar como achar este módulo do produto vetorial entre MP e NQ?Carlos Victor [EMAIL PROTECTED] wrote:
Olá Jefferson ,1)Sejam M,N,P eQ os baricentros , respectivamente. M= (A+B+E)/3 ; N = ( B+C+E)/3 ; P = ( C+D+E)/3 e Q = (A+D+E)/3 . Logo MP = (AC+BD)/3 e NQ = (BD - AC)/3 ; onde AC é o vetor AC .Faça o produto vetorial de MP e NQ e, conclua que o módulo deste produto vetorial é justamente 2/9 do módulo do produto vetorial de AC com BD . ok ? []´s Carlos VictorAt 14:42 19/7/2004, Jefferson Franca wrote:
Duas questões que estão tirando o meu sossego são:01. ABCDE é um pentágono convexo . Mostre que os baricentros dos triângulos ABE, BCE, CDE e DAE formam um quadrilátero convexo cuja área é 2/9 da área do quadrilátero ABCD.,Yahoo! Mail agora ainda melhor: 100MB, anti-spam e antivírus grátis! 
		Yahoo! Mail agora ainda melhor: 100MB, anti-spam e antivírus grátis!

Re: [obm-l] Vetor/EN

2004-07-19 Por tôpico Jefferson Franca
Desculpe, mas vc acha que as duas diagonais de um paralelogramo têm o mesmo comprimento?Pois, foi isso que vc quis dizer quando na sua solução disse que o módulo da diferença é igual ao da soma! 
Ao invés de dizer paralelogramo vc não deveria ter dito retângulo ou quadrado, até pq o quadrilátero formado pelos vetores v e w tem ângulos internos iguais a 90°?[EMAIL PROTECTED] wrote:
Desculpem por não colocar assunto na mensagem anterior e de naum colar aquestão da EN!!!Boa Noite =Instruções para entrar na lista, sair da lista e usar a lista emhttp://www.mat.puc-rio.br/~nicolau/olimp/obm-l.html=
		Yahoo! Mail agora ainda melhor: 100MB, anti-spam e antivírus grátis!

Re: [obm-l] Escola Naval 2004

2004-07-18 Por tôpico Jefferson Franca
Como w e v são perpendiculares, então os prováveis quadriláteros formados por esses vetores só poderiam ser o quadrado ou o retângulo, de qualquer maneira o vetor que representa a soma de w v tem o mesmo tamanho que o vetor que representa o vetor diferença, ou seja, o módulo da diferença entre w e v é o módulo do vetor uIgor Castro [EMAIL PROTECTED] wrote:




(sem as setinhas de vetor):
U = (2,1,-3)
P= (3,-1,0)
Seja V=(a,b,c) e W=(d,e,f)
W e V são perpendiculares -  ad + be + cf = 0 (produto escalar=0)
2 = a+d - 4 = a^2 + 2ad + d^2
1 = b^2 + 2be + e^2
 9 = c^2 +2cf + f^2
- 14 =a^2 + b^2 + c^2 + d^2 + e^2 + f^2+ 2(ad + be + cf) = a^2 + b^2 + c^2 + d^2 + e^2
V - W = (a - d, b - e, c - f) -  |V - W| = sqrt2(a^2 + b^2 + c^2 + d^2 + e^2 + f^2 - 2(ad + be + cf) =
sqrt2(a^2 + b^2 + c^2 + d^2 + e^2 + f^2) = sqrt2(14) (letra B)
Avisem se tiver algo errado..
[]´s
Igor

- Original Message - 
From: João Vitor 
To: [EMAIL PROTECTED] 
Sent: Sunday, July 18, 2004 8:12 PM
Subject: Re: [obm-l] Escola Naval 2004

Essa é de Vetores

 -- - - -- - - --- -
Sabendo q: U = 2i + j - 3k ; U = V + W onde V é paralelo a P = 3i - j e W é

 - - -
perpendicular a P ; Podemos Afirmar q |V - W| é:

A) Sqrt(19)/2

B) Sqrt(14)

C) Sqrt(27)/4

D) Sqrt(20)

E) Sqrt(53)/2

Essa caiu ano passado na Escola Naval!



João Vitor, Fortaleza - CE

- Original Message - 
From: Robÿe9rio Alves 
To: [EMAIL PROTECTED] 
Sent: Saturday, July 17, 2004 8:03 PM
Subject: [obm-l] Probleminha legal, como resolver ?

Sabendo que um balaio de ovo foi dividido entre três pessoas. O primeiro ficou com a metade da quantidade de ovos mais meio ovo. O segundo ficou com a metade do que sobrou mais um muio. Por conseguinte, o último com a metade do que sobrou mais um meio. Pergunta - se 
a) Quantos ovos ( inteiros ) há no balaio ?
b) Quantos ovos ficou a primeira pessoa ?
c) Quantos ovos ficoua segunda ?
d) Quantos ovos ficoua terceira ?
__Do You Yahoo!?Tired of spam? Yahoo! Mail has the best spam protection around http://mail.yahoo.com __Do You Yahoo!?Tired of spam?  Yahoo! Mail has the best spam protection around http://mail.yahoo.com 

[obm-l] elipse

2004-05-29 Por tôpico Jefferson Franca
Será q alguém poderia me ajudar com a questão: Determine a equação das tangentes à elipse (x^2)/4 + (y^2) = 1, que passam pelo ponto P(3,2). 
Yahoo! Mail - Participe da pesquisa global sobre o Yahoo! Mail. Clique aqui!

Re: [obm-l] Re:Provas do C.N

2004-05-19 Por tôpico Jefferson Franca
Leandro, será q vc tbm poderia enviar pra mim estas apostilas? Eu dou aula pra alguns alunos q vão fazer a prova do cn e seria interessante este material! 
Eu preciso dar meu endereço p vc e tentar descobrir como pagar as despesas postais. 
Se vc puder enviar estas provas eu agradeceria. 
Um abraçoleandro-epcar [EMAIL PROTECTED] wrote:
Velho ,vou te mandar uma xerox do livro do manuel "colegio naval 52 a 66 " , mas como voçê vai pagar as despesas,NÃO ENTENDO COMPO FUNCIONA O CORREIOLEANDRO GERALDO DA COSTA COLEGIO:ESCOLA TECNICA PANDIA CALOGERAS BARRA MANSA TEL :024 33261039-- Início da mensagem original ---De: "Alves Dias" [EMAIL PROTECTED]Para: [EMAIL PROTECTED]Cc: Data: Tue, 18 May 2004 10:15:25 -0300Assunto: Provas do C.N Caro Leandro, eu gostaria de obter as provas de matematica do colegio naval que vc tem, obrigado. Jose Aurimenes alves Dias Rua General Roca 460 casa 09 apt. 101 Tijuca RJ CEP. 20521-070  obs. pagarei as despesas! AURI __Acabe com aquelas
 janelinhas que pulam na sua tela.AntiPop-up UOL - É grátis!http://antipopup.uol.com.br/=Instruções para entrar na lista, sair da lista e usar a lista emhttp://www.mat.puc-rio.br/~nicolau/olimp/obm-l.html=Yahoo! Messenger - Fale com seus amigos online. Instale agora!

Re: [obm-l] Re:Provas do C.N

2004-05-19 Por tôpico Jefferson Franca
Não se preocupe que vou fazer de tudo para tornar essas provas públicas. Como vc pode enviar essas provas? O q é preciso?leandro-epcar [EMAIL PROTECTED] wrote:
Mando sem problema nenhum ,é só me garantirem que o material se tornará publico por meio da internet ou outros meios e me adiantarem a postagem.gostaria de eu mesmo torna publico esta 'preciosa'mas como tambem estou me preparando não tenho tento para digita-las-- Início da mensagem original ---De: [EMAIL PROTECTED]Para: [EMAIL PROTECTED]Cc:  Data: Wed, 19 May 2004 13:34:32 -0300 (ART)Assunto: Re: [obm-l] Re:Provas do C.N Leandro, será q vc tbm poderia enviar pra mim estas apostilas? Eu dou aula pra alguns alunos q vão fazer a prova do cn e seria interessante este material!  Eu preciso dar meu endereço p vc e tentar descobrir como pagar as despesas postais.  Se vc puder enviar estas provas eu agradeceria.  Um abraço  leandro-epcar
 <[EMAIL PROTECTED]>wrote: Velho ,vou te mandar uma xerox do livro do  manuel "colegio naval 52 a 66 " , mas como voçê vai  pagar as despesas,NÃO ENTENDO COMPO FUNCIONA O CORREIO  LEANDRO GERALDO DA COSTA  COLEGIO:ESCOLA TECNICA PANDIA CALOGERAS  BARRA MANSA  TEL :024 33261039  -- Início da mensagem original ---  De: "Alves Dias" [EMAIL PROTECTED] Para: [EMAIL PROTECTED] Cc:  Data: Tue, 18 May 2004 10:15:25 -0300 Assunto: Provas do C.N   Caro Leandro, eu gostaria de obter as provas de  matematica do colegio naval que vc tem, obrigado.  Jose Aurimenes alves Dias  Rua General Roca 460 casa 09 apt. 101  Tijuca RJ  CEP. 20521-070obs. pagarei as despesas!  AURI   
 __ Acabe com aquelas janelinhas que pulam na sua tela. AntiPop-up UOL - É grátis! http://antipopup.uol.com.br/= Instruções para entrar na lista, sair da lista e usar a lista em http://www.mat.puc-rio.br/~nicolau/olimp/obm-l.html =   - Yahoo! Messenger - Fale com seus amigos online. Instale agora!___Acabe com aquelas janelinhas que pulam na sua tela.AntiPop-up UOL - É grátis!http://antipopup.uol.com.br/=Instruções
 para entrar na lista, sair da lista e usar a lista emhttp://www.mat.puc-rio.br/~nicolau/olimp/obm-l.html=Yahoo! Messenger - Fale com seus amigos online. Instale agora!

Re: [obm-l] esta indo ou não

2004-05-19 Por tôpico Jefferson Franca
Leandro, eu não recebi nenhuma prova. Vc pode enviar? Que tal tentar scannear? Será que vai ficado pesado?leandro-epcar [EMAIL PROTECTED] wrote:
Já mandei 6 provas do colegio naval e ninguem me deu retorno .Se alguém recebeu alguma prova me comunique..Valeu!!!__Acabe com aquelas janelinhas que pulam na sua tela.AntiPop-up UOL - É grátis!http://antipopup.uol.com.br/=Instruções para entrar na lista, sair da lista e usar a lista emhttp://www.mat.puc-rio.br/~nicolau/olimp/obm-l.html=Yahoo! Messenger - Fale com seus amigos online. Instale agora!

[obm-l] Re:

2004-03-14 Por tôpico Jefferson Franca
Melhorei sim, obrigado pela preocupação! Tenho as provas , mas no momento não posso enviá-las espere até segunda. 
Um abraçoViviane Silva [EMAIL PROTECTED] wrote:



Oi professor, espero que tenha melhorado.
Professor você tem as provas do ime?

MSN Messenger: converse com os seus amigos online. Instale grátis. Clique aqui. = Instruções para entrar na lista, sair da lista e usar a lista em http://www.mat.puc-rio.br/~nicolau/olimp/obm-l.html =Yahoo! Mail - O melhor e-mail do Brasil. Abra sua conta agora!

Re: [obm-l] Interessante

2004-02-28 Por tôpico Jefferson Franca
Vc tentou com expoente ímpar? Tá certo com expoente par, ou seja, o final é 76, porém, com expoente ímpar o final é sempre 24!Fabio Contreiras [EMAIL PROTECTED] wrote:




Acho que quando é expoente par termina com 76 certo?

- Original Message - 
From: Jefferson Franca 
To: obm 
Sent: Saturday, February 28, 2004 3:55 PM
Subject: [obm-l] Interessante



Ontem um amigo mostrou que uma potência de base 24 e expoente ímpar e natural sempre dá como resultado um número que termina em 24! Pq?


Yahoo! Mail - O melhor e-mail do Brasil. Abra sua conta agora! 


Esta mensagem foi verificada pelo E-mail Protegido Terra.Scan engine: VirusScan / Atualizado em 27/02/2004 / Versão: 1.4.1Proteja o seu e-mail Terra: http://www.emailprotegido.terra.com.br/ 


E-mail classificado pelo Identificador de Spam Inteligente.Para alterar a categoria classificada, visite http://www.terra.com.br/centralunificada/emailprotegido/imail/imail.cgi?+_u=fabiocontreiras_l=1077995594.173729.28919.conventos.terra.com.br Yahoo! Mail - O melhor e-mail do Brasil. Abra sua conta agora!

Re: [obm-l] Interessante

2004-02-28 Por tôpico Jefferson Franca
Desculpe , mas 24^2 é igual a 76? e se n for ímpar 76^n = 76 continua valendo?Fábio Dias Moreira [EMAIL PROTECTED] wrote:
-BEGIN PGP SIGNED MESSAGE-Hash: SHA1Jefferson Franca <[EMAIL PROTECTED]>said: Ontem um amigo  mostrou que uma potência de base 24 e expoente ímpar e natural sempre dá como resultado um número que termina em 24! Pq? [...](mod 100), 24^(2*n+1) = 24 * (24^2)^n = 24 * 76^n. Como 76^2 = 76 (mod 100), não é difícil ver que 76^n = 76. Logo 24^(2*n+1) = 24 * 76 = 24.[]s,- -- Fábio "ctg \pi" Dias Moreira-BEGIN PGP SIGNATURE-Version: GnuPG v1.2.3 (GNU/Linux)iD8DBQFAQO6+alOQFrvzGQoRAkGHAKCOQNXNPu7Ns0V91jhJUMRdqnR9BACeL/1Ut+9qwJTmHuYiZLn2ucEuJc8==GnNp-END PGP SIGNATURE-=Instruções para entrar na lista, sair da lista e usar a lista
 emhttp://www.mat.puc-rio.br/~nicolau/olimp/obm-l.html=Yahoo! Mail - O melhor e-mail do Brasil. Abra sua conta agora!

[obm-l] Numero de Napier

2004-02-09 Por tôpico Jefferson Franca
Será q alguém sabe como calcular o valor numérico do número de Napier n, mas sem usar Taylor ?Yahoo! GeoCities: 15MB de espaço grátis para criar seu web site!

Re: [obm-l] irracionais

2004-02-04 Por tôpico Jefferson Franca
Valeu Domingos eu achei animal a solução"Domingos Jr." [EMAIL PROTECTED] wrote:
suponha sqrt(15) + sqrt(10) - sqrt(6) - 3 racional;então (1) = sqrt(15) + sqrt(10) - sqrt(6) é racional =(1)² é racional =15 + 10 + 6 + 2sqrt(150) - 2sqrt(60) - 2sqrt(90) é racional =sqrt(150) - sqrt(60) - sqrt(90) é racional =(2) = 5sqrt(6) - 2sqrt(15) - 3sqrt(10) é racional ...de (1) e (2) temos(1)*2+(2) = 3sqrt(6) - sqrt(10) é racional[3sqrt(6) - sqrt(10)]² é racional =9*6 + 10 - 6sqrt(60) é racional =sqrt(60) = 2sqrt(15) é racional =sqrt(15) é racionalsqrt(15) = a/b com a, b uma fração irredutível15 = a²/b²15b² = a² = 3, 5|a = a = 3*5*k = a² = 15²k²15b² = 15²k²b² = 15k² = 15|b² e portanto mdc(a, b) = 15, e está aí a nossacontradição.PS: de forma geral, se n é inteiro e não é quadrado perfeito sqrt(n) éirracional.- Original Message
 ----- From: "Jefferson Franca" <[EMAIL PROTECTED]>To: <[EMAIL PROTECTED]>Sent: Tuesday, February 03, 2004  12:43 PMSubject: [obm-l] irracionaisComo consigo provar que sqrt(15) + sqrt(10) - sqrt(6) - 3 é irracional?-Yahoo! GeoCities: 15MB de espaço grátis para criar seu web site!=Instruções para entrar na lista, sair da lista e usar a lista emhttp://www.mat.puc-rio.br/~nicolau/olimp/obm-l.html=Yahoo! GeoCities: 15MB de espaço grátis para criar seu web site!

[obm-l] irracionais

2004-02-04 Por tôpico Jefferson Franca
Será q alguém pode indicar algum livro ou nota de aula que esteja na net sobre irracionais e racionais?Yahoo! GeoCities: 15MB de espaço grátis para criar seu web site!

  1   2   >